PR- Final

अब Quizwiz के साथ अपने होमवर्क और परीक्षाओं को एस करें!

A lawyer is interested in obtaining legal business from a mineworkers' union that has many members in the state in which the lawyer practices. As a result of a recent mine fire and explosion in which several union members were killed, the union has succeeded in persuading the appropriate state agency to bring an administrative action against the company that owns the mine for failing to install smoke detectors, which might have saved some lives in the disaster. Although the lawyer is in no way involved in the case, he sees this as an opportunity to obtain future business from the union by showing the union that he is strongly on its side in the mine disaster case. The lawyer telephones a popular call-in radio show and says that he is shocked and appalled at the callousness of the mining company that caused the recent disaster in which so many miners were killed. He further opines that the mining company was willful and wanton in its failure to install smoke detectors, and expresses hope that the company will not be allowed to escape the consequences of its despicable conduct. Without the lawyer's knowledge or consent, his statement was later printed in several news-papers in the state. Is the lawyer subject to discipline for his conduct? (A) Yes, because he was substantially motivated by his desire to attract fee-paying business. (B) Yes, because lawyers must not make public comments concerning pending litigation. (C) No, because he did not make any false or misleading claims about himself or his services. (D) No, because the statement was printed in the newspapers without his knowledge or consent.

C The lawyer is not subject to discipline because he did not make any false or misleading claims about himself or his services. Lawyers, like other citizens, have the right to express their views in the media on newsworthy issues. Even if a lawyer's sole purpose in seeking media publicity is to lure clients, the state may not impose professional discipline on the lawyer absent a compelling state interest. A lawyer who uses the media to lure clients may, however, be disciplined for making statements or claims that are false or misleading about the lawyer or his services. [ABA Model Rule 7.1] Here, the lawyer made no statements about himself or his services other than the fact that he is a lawyer and his opinion about the incident. There is nothing false or misleading in his communication. (A) is wrong because, as discussed above, the fact that the lawyer was motivated by the desire to attract fee-paying business is irrelevant. This is not a case of in-person or live electronic solicitation, which is the only circumstance in which this consideration is relevant. (B) is wrong because it is overbroad. Lawyers can and do make public statements about pending litigation all the time. Lawyers who are involved in a proceeding cannot make statements that they know will have a substantial likelihood of materially prejudicing an adjudicative proceeding. [ABA Model Rule 3.6(a)] That is not the case here; the lawyer has no reason to believe his opinion will materially prejudice the state agency bringing the action. (D) is wrong because the fact that the statement was printed in the newspapers is of absolutely no consequence; it does not affect the propriety of the lawyer's behavior regardless of whether he gave consent.

A law student is applying to become a member of the state bar. The bar application questionnaire asks whether the applicant has ever used any illegal drug in violation of state law. When the law student was in high school, she occasionally smoked marijuana, which is a misdemeanor under state law. However, the statute of limitations has run on these incidents, and the law student believes that she could not validly be kept out of the state bar for those offenses. She therefore believes that the question is irrelevant and an invasion of her privacy in violation of the state constitution. She fears, however, that challenging the question could brand her as a troublemaker and delay her admission to the bar. Which of the following would be proper? (A) Answer the question in the negative, without saying more. (B) Answer the question in the negative, citing the state constitution's privacy provision. (C) Decline to answer the question, citing the state constitution's privacy provision. (D) Decline to answer the question, citing the federal constitutional privilege against self-incrimination.

C While a bar applicant must cooperate in reasonable investigations by the state bar and make disclosures relevant to her fitness to practice, she may challenge the validity of a question on legally tenable grounds. [See ABA Model Rule 3.4(c)] That a question violates the state constitution would be a legally tenable argument. Moreover, by declining to answer, the law student has not made an untrue statement. (A) and (B) would not be proper because a bar applicant must not make untrue statements on a bar application. [ABA Model Rule 8.1(a)] If the law student answers in the negative, she will have made an untrue statement. (D) would not be proper because the self-incrimination privilege is inapplicable here. Even if the privilege against self-incrimination applies to questions on bar applications (a debatable proposition), it does not apply when criminal punishment is barred by the statute of limitations.

A client made a preliminary contact with a lawyer to see if she wanted to hire the lawyer to defend her in a tort case that had been assigned to a judge. The lawyer told the client that the initial consultation was free of charge. After listening to the client's brief outline of the case, the lawyer told her that she knew how to get a favorable decision from that judge. She said that he would be running for re-election 18 months from now, and he would need money for his campaign. She advised the client to send him a $2,000 campaign contribution now, with a nice note wishing him well in his bid for re-election. She explained that the judge's opponent in the election would be a local lawyer, and that although the local lawyer was an honest fellow, his two brothers were associated with organized crime. The lawyer offered to write a guest editorial for the local paper, praising the judge's judicial record and implying that the local lawyer was a crook. With the contribution and the letter, the lawyer said she thought they could count on the judge to reach a wise decision in the client's case. The client hired the lawyer and sent the judge the $2,000. The lawyer wrote the guest editorial, and it was published in the local paper. For which of the following is the lawyer not subject to discipline? (A) Saying that she knew how to get a favorable ruling from the judge. (B) Advising the client to send the judge a campaign contribution. (C) Writing the guest editorial. (D) Accepting the case after giving free legal advice.

D The lawyer is not subject to discipline for (D) because she was not offering unsolicited free legal advice, then accepting employment arising from it. [See ABA Model Rule 7.3] The client initiated the contact, asked for the advice, and was in the midst of negotiating the lawyer's employment when the advice was given. On the other hand, a lawyer must not state or imply that she has the ability to improperly influence a government official or that she can achieve results by means that violate the Rules of Professional Conduct or other law. [ABA Model Rule 8.4(e)] Thus, the lawyer is subject to discipline for (A), saying that she knows how to get a favorable ruling from the judge, which implies that she can influence the judge. Advising the client to make the campaign contribution can be viewed in two ways, both of which subject the lawyer to discipline. If the $2,000 is viewed harshly as a bald bribe, then the lawyer is subject to discipline for counseling a client to commit a crime. [ABA Model Rule 1.2(d)] If the $2,000 is viewed benignly as a campaign contribution, the lawyer is subject to discipline for assisting a judge to violate a judicial ethics rule. [ABA Model Rule 8.4(f)] The Code of Judicial Conduct prohibits the judge from personally accepting campaign contributions. Contributions must be given to a campaign committee. [CJC Rule 4.1(A)(8)] Thus, (B) subjects the lawyer to discipline. The lawyer is subject to discipline for (C) because the lawyer apparently believes that the local lawyer running against the judge is an "honest fellow," yet her editorial implies that the local lawyer is a crook. A lawyer is prohibited from making a statement about a judicial candidate, either knowing that the statement is false or with reckless disregard as to its truth or falsity. [ABA Model Rule 8.2(a)]

A solo practitioner who does municipal bond work in a single state is nearing retirement age and takes on a young attorney as a partner. Their partnership agreement provides that the solo practitioner will train the young attorney and will receive 75% of the partnership's net earnings during the first three years, and that the young attorney will receive the remaining 25%. The agreement also provides that if the young attorney leaves the partnership before the end of the first three years, he will remit to the solo practitioner 75% of all fees he earns thereafter from municipal bond work in the state. Finally, the agreement provides that if the solo practitioner and the young attorney are still partners when the solo practitioner retires, the young attorney will pay the solo practitioner retirement benefits of $3,000 per month until the solo practitioner's death; in return, upon his retirement, the solo practitioner will turn over to the young attorney all of the partnership assets (including goodwill) and will not thereafter practice municipal bond law in the state. Are the solo practitioner and the young attorney subject to discipline for entering into this partnership agreement? (A) No, because the agreement gives the solo practitioner retirement payments in return for the restriction on his right to practice. (B) No, because the agreement enables the solo practitioner to sell the partnership assets in return for the restriction on his right to practice. (C) Yes, because of the restriction on both the solo practitioner's and the young attorney's right to practice. (D) Yes, because of the restriction on the young attorney's right to practice if he leaves the partnership within the first three years.

D The solo practitioner and the young attorney are subject to discipline for entering into the partnership agreement because of the restriction on the young attorney's right to practice. A lawyer must neither make nor offer a partnership agreement that restricts a lawyer's right to practice after termination of the relationship, except for an agreement concerning benefits upon retirement. [ABA Model Rule 5.6(a)] Under the agreement here, if the young attorney leaves the partnership within the first three years, he must pay the solo practitioner 75% of the fees he earns thereafter from municipal bond work he does in the state. This provision unduly restricts the young attorney's right to practice. (A) is wrong because it overlooks the improper restriction on the young attorney's right to practice. (B) is wrong because it overlooks the restriction on the young attorney. Furthermore, it implies that the sale of assets in exchange for the restriction on the solo practitioner is necessary to restrict the solo practitioner's right to practice; it is not. (C) is wrong because the restriction on the solo practitioner's right to practice is properly incident to the retirement benefits he will receive from the young attorney.

A prospective client comes to a law office seeking a lawyer to defend him in a civil action for aggravated assault and battery. A lawyer agrees to talk preliminarily with the client, just to obtain enough background information to decide whether she can defend him. The client explains that he has an alcohol problem; indeed, he gets roaring drunk about three nights a week. On the night in question, the client said that a loud-mouthed stranger in his neighborhood tavern made a derogatory comment about the client's favorite basketball team. The client responded by "tapping" the stranger over the head with a pool cue, not once but four times. At that point, the lawyer suddenly realizes that the client must be the rotten husband in the hotly disputed divorce and child custody case in which her law partner is representing the aggrieved wife. The lawyer stops the client and tells him that she cannot defend him in the assault and battery case because of her partner's work for the client's wife. Which of the following is true? (A) The partner must withdraw from representing the wife because the lawyer has received confidential information from the client that would be harmful to the client if used in the divorce and child custody case (B) It would be proper for the partner to represent the wife and for the lawyer to represent the client in the assault and battery case because the two matters are not substantially related. (C) The partner may continue representing the wife, but only if the wife gives informed consent, confirmed in writing. (D) The partner may continue representing the wife if the lawyer is screened off from participation in the case and obtains no part of the fee in the case, and if the firm promptly sends the client written notice of the situation.

D This question is governed by ABA Model Rule 1.18, which concerns duties to a prospective client. The information that the lawyer obtained about the prospective client's alcohol abuse and his violent response to the stranger's comment could be harmful to the prospective client if the wife uses it to help prove that the couple should be divorced and that the prospective client should not be given custody of their children. Therefore, the lawyer herself could not represent the wife in the divorce and child custody case. [See ABA Model Rule 1.18(c)] The lawyer's disqualification is imputed to her law partner. [Id.] However, if the conditions mentioned in (D) are satisfied, then her law partner may continue representing the wife. [See ABA Model Rule 1.18(d)(2)] (C) is wrong because it calls for informed consent by the wife only. ABA Model Rule 1.18(d)(1) would require informed consent, confirmed in writing, from both the prospective client and the wife. (B) is wrong because the two matters are "substantially related" [ABA Model Rule 1.18(c)] in that use of the information that the prospective client disclosed to the lawyer could be harmful to the prospective client if used in the divorce and child custody case. (A) is wrong because it ignores the two possible ways that would allow the law partner to continue representing the wife. [ABA Model Rule 1.18(d)(1), (2)]

An attorney was representing the plaintiff at a bench trial of a civil action pending before a judge. Midway through the plaintiff's case-in-chief, the judge called the attorney into his chambers. The judge told the attorney that he thought the attorney's case was very weak, but that he could be mistaken because he was distracted by money troubles. The judge went on to say that if he could get a $50,000 loan, he would feel much better. The attorney responded that he would be happy to loan the judge $50,000 to help him out as a friend. Later that afternoon, a messenger delivered an envelope containing $50,000 in cash to the judge's chambers. No mention was made of a promissory note, a repayment date, or an interest rate. Two days later, the plaintiff settled his lawsuit so the judge never had to decide the case. Three months later, the judge repaid the $50,000 to the attorney, together with interest at the market rate. Is the attorney subject to criminal liability for lending the money to the judge? (A) Yes, if it is proven that the judge intended to induce the attorney to make the loan in return for a decision in favor of the plaintiff. (B) Yes, if it is proven that, in making the loan, the attorney intended to induce the judge to decide the case in favor of the plaintiff. (C) No, because as the matter turned out, the judge never had to decide the plaintiff's case. (D) No, because the judge repaid the loan with interest.

B The attorney is subject to criminal liability if he intended to induce the judge to decide the case in the plaintiff's favor. The common law crime of bribery consists of the corrupt payment or receipt of anything of value in return for official action. The $50,000 loan was obviously a thing of value. Thus, if the attorney intended the loan as an inducement to the judge to decide the case in favor of the plaintiff, then the attorney is guilty of bribery. (A) is wrong because in deciding whether the attorney is subject to criminal liability, it is the attorney's intent that counts, not the judge's intent. (C) is wrong because the attorney's crime was complete when he gave the loan, even though the judge never had to decide the case. (D) is wrong because bribery does not require an outright gift; a $50,000 loan is a thing of value, especially a loan with no repayment date, no promissory note, and no interest specified.

A retired lawyer practiced admiralty and maritime law for 45 years in Maine. He stopped paying his bar dues in Maine when he retired, and he is no longer licensed to practice there. He and his wife moved to a retirement village in New Mexico, but he did not seek to become licensed to practice law in New Mexico. After a few months of playing golf and puttering in the garden, the retired lawyer got bored and started missing the challenges of law practice. He therefore joined the unpaid staff of volunteer lawyers at the Rio Grande Walk-In Legal Advice Clinic, which is run by a nonprofit organization. The clinic's purpose is to offer free, quick, accurate, compassionate legal advice to walk-in clients who cannot afford ordinary legal service and who have legal problems that can be solved quickly, without litigation or other time-consuming procedures. Before they ever see one of the clinic's lawyers, all of the clients must give informed consent to the limited nature of the legal services they will receive. The retired lawyer works at the clinic three days a week, and he dispenses legal advice on all sorts of matters—although he has yet to find a client who needed admiralty or maritime advice. The retired lawyer enjoys the work because it makes him feel useful again, and because it gives him a cornucopia of interesting stories to tell his wife about his clients' various legal troubles. Which of the following statements is correct? (A) The retired lawyer is subject to discipline for practicing law without a license. (B) The retired lawyer is subject to discipline for failing to pay his bar dues in Maine. (C) The retired lawyer's volunteer work is proper because one does not need to be licensed to dispense legal advice at a quick-service clinic like this one. (D) The retired lawyer's conversations with his wife are proper because no confidential lawyer-client relationship is formed at a quick-service clinic like this one.

A (A) is correct because the retired lawyer's dispensation of legal advice constitutes the "practice of law," and he is doing it in a jurisdiction where he is not licensed. [See ABA Model Rule 5.5(a)] (B) is incorrect because the retired lawyer is under no obligation to pay bar dues in a state where he no longer lives or practices. (C) is incorrect because one does need to be licensed in order to dispense legal advice at a walk-in legal clinic. [See ABA Model Rule 6.5, which loosens the conflict of interest rules for clinic lawyers but does not authorize them to practice without a license] A law student or similar unlicensed person can work at such a clinic under the close supervision of a lawyer, but the question makes no mention of the retired lawyer's work being supervised. [See comment 1 to ABA Model Rule 5.3] (D) is incorrect because there is a lawyer-client relationship between the retired lawyer and his walk-in clients. [Comment 1 to ABA Model Rule 6.5] One element of that relationship is the lawyer's duty of confidentiality [ABA Model Rule 1.6], and the retired lawyer breaches that duty when he tells his wife about his clients' legal troubles.

An attorney limits her practice to family law. A married couple came to her, hoping to save their marriage. After hearing their story, the attorney explained that she could act as a third-party neutral between them—not representing either one of them but facilitating their efforts to work through their difficulties. The attorney explained that neither of them would have the protections afforded by an ordinary attorney-client relationship, such as the attorney-client privilege, and both said that they understood. The attorney held a series of meetings with the couple (sometimes with both, and other times with just one). The attorney began each meeting with a reminder that, in the event of later litigation between the husband and wife, the attorney-client privilege would not protect what was said at the meeting. At one of the attorney's individual meetings with the husband, he disclosed that he occasionally used drugs and sometimes used the family's savings to purchase them. Ultimately, the husband and wife were unable to resolve their differences, and the wife sued the husband for a divorce and for custody of their three children. At the custody hearing, the wife's lawyer called the attorney to the witness stand and asked what the husband told her about his drug use. The attorney refused to answer, citing the attorney-client privilege. Which of the following is correct? (A) The privilege claim should be overruled; if the attorney refuses to answer, she would be subject to litigation sanction. (B) The privilege claim should be sustained; if the attorney reveals what the husband said, she would be subject to discipline. (C) The attorney is subject to discipline for attempting to serve as a third-party neutral in a family law matter. (D) The attorney is subject to civil liability for attempting to serve as a third-party neutral in a family law matter.

A (C) is incorrect because there is no disciplinary rule against a lawyer acting as a third-party neutral in a family law matter, and it was proper for the attorney to undertake the neutral's role in helping the husband and wife try to resolve their marital problems. [See Restatement §130, comment d] (D) is incorrect because the facts contain nothing to support a civil claim against the attorney for the work she did as a third-party neutral. (B) is incorrect because when two parties jointly consult an attorney on a matter of common interest, neither of them can claim the attorney-client privilege in subsequent civil litigation between them. [See Restatement §75] (A) is correct because the husband cannot claim the privilege for the reason stated above. If the judge overrules the privilege claim and the attorney nevertheless refuses to answer the question, she can be sanctioned for contempt of court.

A judge presides over a state trial court. Every six years, trial judges in the state must stand as candidates in a partisan public election to determine whether they will retain their positions. The judge will be a retention candidate in the election to be held nine months from now. In that same election, the judge's husband, an attorney, will be a candidate for lieutenant governor of the state. The state allows judicial candidates to form campaign committees up to a year before the election. Which of the following may the judge do? (A) Establish a campaign committee that will solicit reasonable contributions for the judge's campaign. (B) Publicly endorse her husband as a candidate for lieutenant governor. (C) Attend political gatherings in the company of her husband, and speak on behalf of both herself and him. (D) Personally solicit contributions to her own campaign.

A A judge may establish a campaign committee no earlier than an amount of time prior to the election set by the jurisdiction. [CJC Rule 4.2(B)(1)] Because the state allows campaign committees to be formed up to one year before the election, and it is nine months before the election under the facts given, (A) is proper. (B) is improper; a judge may not publicly endorse a candidate for another public office. [CJC Rule 4.1(A)(3)] (C) is improper; the judge may attend a political gathering in the company of her husband, and she may speak on her own behalf, but she must not speak on his behalf. [CJC Rules 4.1(A)(3), 4.2(B)] (D) is improper; a judge is prohibited from personally soliciting contributions for her own campaign. [CJC Rule 4.1(A)(8)]

A lawyer represented a plaintiff in a personal injury case. While on his way home after the first day of the trial, the lawyer stopped for dinner at the pub down the street from the courthouse. A juror walked in, sat at the barstool next to the lawyer, and complimented the lawyer on his opening statement. In an effort to be polite and without any intention of influencing the juror, the lawyer replied, "Thanks. I did my best to explain the scope of my client's injuries—she's had a really hard time since the accident." The lawyer then excused himself and moved to a table. The jury eventually returned a verdict in favor of the plaintiff. Is the lawyer subject to discipline? (A) Yes, because the lawyer spoke with the juror outside of the official proceedings (B) Yes, because the jury returned a verdict for the plaintiff. (C) No, because the lawyer did not intend to influence the juror. (D) No, because the juror initiated the conversation.

A A lawyer must not communicate ex parte with a juror unless authorized by law or court order. [ABA Model Rule 3.5(b)] Communicating with a juror is generally improper even if the subject matter is unrelated to the case. [See Restatement §115] Here, the lawyer's remark about his client's injuries was a clear violation

A solo practitioner limits her law practice to two kinds of cases: medical malpractice and products liability. When the solo practitioner turned 67, she began looking for someone to buy her law practice. Ultimately, she sold the medical malpractice part of her practice to lawyer Alpha for $400,000, and she sold the products liability part of her practice to attorney Beta for $250,000. The solo practitioner then retired. Within 30 days after the sales to Alpha and Beta, approximately 40% of the solo practitioner's former clients decided to collect their case files and take their business to different lawyers. Were the sale from the solo practitioner to Alpha and the sale from the solo practitioner to Beta proper? (A) Yes, even though the solo practitioner sold pieces of her practice to two different buyers, and even though 40% of the solo practitioner's clients left the buyers within 30 days. (B) Yes, but the solo practitioner is subject to civil liability to the two buyers for unjust enrichment because 40% of the solo practitioner's clients left the buyers within 30 days. (C) No, because the solo practitioner sold pieces of her practice to two different buyers. (D) No, because 60% of the solo practitioner's clients stayed with the buyers for more than 30 days.

A ABA Model Rule 1.17 permits a lawyer to sell her entire law practice, or an area of her law practice, to one or more lawyers or law firms. Here, the solo practitioner sold her entire law practice to two different lawyers, and ABA Model Rule 1.17 permits that. The departure of 40% of the solo practitioner's clients does not cause the sales to violate ABA Model Rule 1.17. Indeed ABA Model Rule 1.17(c)(2) requires the selling lawyer to notify her clients that they have a right to pick up their files from the buyer and take them to a different lawyer. [See also comment 2 to ABA Model Rule 1.17] (B) and (D) are wrong because clients are not like sheep that can be bought and sold, willy-nilly. To hold the solo practitioner either subject to discipline or civilly liable in unjust enrichment would be inconsistent with the clients' right to pick up their files and take them to a different lawyer. The solo practitioner, Alpha, and Beta made their sales contracts in the context of ABA Model Rule 1.17 so Alpha and Beta cannot claim that they were surprised when clients departed. (C) is wrong because ABA Model Rule 1.17(b) permits a lawyer to sell her entire practice to one or more lawyers or law firms. One might also argue that the solo practitioner's medical malpractice cases are in a different "area of practice" (professional malpractice) from her products liability cases. But that argument is not necessary here because the solo practitioner sold her entire law practice.

A paralegal works for the law firm of Alpha & Beta. Her direct supervisor is partner Alpha, whose practice is limited to international trade law. Partner Beta is the firm's leading trial lawyer, both in commercial and personal injury cases. On her way to work one morning, the paralegal saw a pedestrian run down in a crosswalk by a speeding car. The paralegal rendered first aid, and while she was waiting with the pedestrian for the ambulance, the paralegal provided the pedestrian with a business card and urged him to call the firm to obtain legal representation in connection with his injuries. When she got to work, she told partner Alpha what she had done. Alpha admonished the paralegal not to hand out the firm's cards in such situations, but he did not discuss the matter with partner Beta. Is Alpha subject to discipline? (A) Yes, because he failed to warn Beta not to take the pedestrian's case. (B) Yes, because as the paralegal's supervisor, he is responsible for any unethical act she commits. (C) No, because as a nonlawyer, the paralegal is free to recommend a lawyer to someone if she wishes. (D) No, because the paralegal may not have been aware at the time that she did anything wrong.

A Alpha is subject to discipline for failing to warn Beta not to take the case. If the paralegal were a lawyer, her conduct would violate ABA Model Rule 7.3(a), which prohibits in-person solicitation. The partners in a firm are responsible for educating their nonlawyer employees about ethics issues and making reasonable efforts to assure that those employees comply with ethics rules. [ABA Model Rule 5.3(a)] Moreover, a partner is subject to discipline if he learns about the violation of an ethics rule by a nonlawyer employee "when its consequences can be avoided or mitigated," but the partner "fails to take reasonable remedial action." In this case, the consequences of the paralegal's solicitation could have been avoided by warning Beta not to take the pedestrian's case. Because he failed to warn Beta, Alpha is subject to discipline. (B) is wrong because it is too broad. A lawyer's responsibility for a nonlawyer employee's ethics violation is limited to situations in which the lawyer orders it, ratifies it, or learns about it in time to remedy it and does not do so. [ABA Model Rule 5.3(c)] (C) is wrong because even though people are generally free to recommend a lawyer to someone else, that does not allow the paralegal to solicit business for the firm that employs her. [ABA Model Rule 8.4(a)] (D) is wrong because Alpha and the other partners in the firm had a duty to educate the paralegal about ethics rules. [ABA Model Rule 5.3(a)] Furthermore, even if the paralegal acted innocently, that does not excuse Alpha's failure to warn Beta not to take the case.

Alpha Corporation ("Alpha") is incorporated in State A and its principal place of business is located there. It manufactures and sells clothing under the trademark Alpha ® and licenses its trademark to garment makers in other states. Beta Clothing Co. ("Beta") is incorporated in State B and has its sole place of business there. Beta hired a lawyer licensed to practice only in State B to accompany Beta's chief buyer on a trip to State A to negotiate a license to use Alpha's trademark. Beta's lawyer will be negotiating with Alpha's lawyer, who is licensed to practice only in State A. Neither lawyer is licensed to practice before the United States Patent and Trademark Office. If Beta's lawyer travels to State A to negotiate the contract, is she subject to discipline? (A) No, because admission to practice in State A was not necessary to negotiate the trademark license there. (B) No, because Alpha's lawyer is licensed to practice in State A. (C) Yes, because the lawyer was not admitted to practice in State A. (D) Yes, because the lawyer was not admitted to practice before the United States Patent and Trademark Office.

A Beta's lawyer is not subject to discipline because admission to practice in State A is not required to negotiate a license there. A lawyer must not practice law in a state where she is not admitted to practice. [ABA Model Rule 5.5(a)] No state, however, would regard the lawyer's conduct as unauthorized practice. The lawyer was admitted in State B, she was representing a State B client, and the trademark license has an important nexus to State B because Beta's manufacturing operations will take place in State B. [See Hazard & Hodes, §46.6] Moreover, ABA Model Rule 5.5(c)(4) permits a lawyer to temporarily practice out of state if that practice is reasonably related to the lawyer's home state practice. Here, the lawyer's going to State A to negotiate a license to use Alpha's trademark is reasonably related to her State B practice of representing Beta.

A potential client consulted an attorney, hoping to hire her to represent him as plaintiff in a medical malpractice action against his doctor. Without mentioning the doctor's name, the client described the alleged acts of malpractice and said that they happened more than two years ago. Only at that point did the potential client mention his doctor's name. The attorney immediately stopped the potential client and said she could not represent him because she was already representing the doctor in an unrelated matter, and she urged him to consult another lawyer. That was the end of the conversation. The potential client did nothing further for 15 months, at which point he consulted another lawyer. By that time, the statute of limitations had run on the potential client's claim against the doctor. The potential client then sued the first attorney for legal malpractice, alleging that the attorney was negligent in not warning him about the statute of limitations. Is the attorney subject to civil liability in the potential client's malpractice case? (A) No, because the attorney did what a reasonably prudent lawyer would do in the circumstances—decline to represent the potential client and suggest that he consult other counsel. (B) No, because the potential client never became the attorney's client and is therefore not a proper plaintiff in a malpractice action against the attorney. (C) Yes, because the attorney had no legal or ethical reason to reject the potential client as a client and therefore had a duty to warn him about the statute of limitations. (D) Yes, because a reasonably prudent lawyer would have foreseen that the potential client might delay in consulting another lawyer.

A Generally speaking, a lawyer must not represent a client in a presently pending piece of litigation and simultaneously oppose that client in a different piece of litigation, without each client's informed consent, confirmed in writing. [See comment 6 to ABA Model Rule 1.7] Here, the attorney may have believed that the conflict of interest that would have been created by his undertaking representation of the prospective client was unconsentable. Thus, he acted properly in declining to represent the prospective client, even though the prospective client's case was unrelated to the one in which the attorney was representing the doctor. However, a lawyer does owe a duty of reasonable care to a prospective client, even though no attorney-client relationship ever comes about. [See Restatement of the Law Governing Lawyers (hereinafter "Restatement") §15] Ordinarily, that duty would include cautioning the prospective client about an impending statute of limitations deadline. [Id.] Here, however, a cautionary word to the prospective client would constitute disloyalty to the existing client, the doctor. [See, e.g., Flatt v. Superior Court, 9 Cal. 4th 275 (1994)—warning prospective client about statute of limitations was not required when it would be disloyal to present client] The attorney therefore acted properly in simply suggesting that the prospective client consult other counsel. (B) is incorrect because a lawyer does owe a duty of reasonable care to a prospective client. (C) is incorrect because the attorney did have a legal and ethical reason to reject the prospective client as a client: the simultaneous representation of the doctor in the unrelated matter. (D) is incorrect because the foreseeability of the harm is not the whole of the analysis. Even if the attorney could foresee that the prospective client would dawdle and let the statute of limitations run, the attorney's duty of loyalty to the doctor required him not to warn the prospective client about the statute of limitations.

A judge serves on a state trial court that has nine other judges. Her husband is a life insurance salesman for a large life insurance company. The life insurance company is occasionally a litigant in the court on which the judge sits. Every year the life insurance company runs a national sales contest in which the person who sells the most life insurance during the year receives a valuable prize. The judge's husband won this year and took the judge on an all-expense-paid vacation in Europe. She did not make a public report of the prize. Was it proper for the judge to allow her husband to accept the prize and take her on the European vacation? (A) Yes, because acceptance of the prize cannot reasonably be perceived as undermining the judge's integrity or impartiality (B) Yes, because the prize was won by her husband, not by the judge. (C) No, because the judge did not make a public report of the prize. (D) No, because the life insurance company may later appear as a litigant in the court on which the judge sits.

A It was proper for the judge to allow her husband to accept the prize because acceptance thereof does not reasonably appear to undermine the judge's integrity or impartiality. A judge may accept benefits associated with her spouse's business activity that incidentally benefit the judge. [CJC Rule 3.13(B)(8)] (B) is wrong because it ignores the general rule on family members' accepting gifts and other benefits. (C) is wrong because such benefits of a spouse are not subject to the public reporting requirement. [CJC Rule 3.13(B)] (D) is wrong because if the prize is proper under CJC Rule 3.13(B)(8), it does not become improper simply because the insurance company may later appear as a litigant in the judge's court.

An attorney is licensed to practice law in State A, but he is not engaged in the active practice of law. The attorney and his non-attorney partner operate a temporary placement service for legal secretaries in State B. The attorney performs most of his work for the agency in State A and is not licensed to practice law in State B. Neither does he hold himself out to be so licensed. An investigation by State B authorities results in the discovery that the attorney and his partner have intentionally filed improper state business tax returns. Is the attorney subject to discipline in State A? (A) Yes, because his actions in State B constitute fraud. (B) Yes, because he supervises the temporary service business from State A. (C) No, because the attorney is not licensed to practice law in State B. (D) No, because this situation does not involve the practice of law.

A The attorney is subject to discipline because his actions in State B constitute fraud. A lawyer is subject to discipline not only for violating a disciplinary rule, but also for committing a criminal act that reflects adversely on his honesty, trustworthiness, or fitness as a lawyer in other respects, or for engaging in conduct involving dishonesty, fraud, deceit, or misrepresentation. [ABA Model Rule 8.4] The type of fraud described clearly falls within this rule. (B) is wrong because a lawyer is subject to discipline in a state where he is licensed to practice even if the misconduct occurred in another jurisdiction. [ABA Model Rule 8.5(a)] (C) is wrong because the fact that the attorney was not licensed in State B is irrelevant. He was not practicing law in State B, so he is not subject to discipline on the ground of unauthorized practice. He is subject to discipline, however, for filing improper tax returns. As discussed above, the attorney cannot escape discipline for his fraudulent conduct because it occurred in another jurisdiction. (D) is wrong because, as discussed above, a lawyer may be disciplined for dishonest conduct regardless of whether it is related to the practice of law.

An attorney represented a defendant in a criminal trial. After the jury returned a guilty verdict, the defendant was taken to jail and the jury was discharged. While walking to his car, the disappointed attorney spotted one of the courtroom spectators in the parking lot. The attorney recalled that the spectator had been a member of the jury pool, but he had exercised a peremptory challenge against her because he instinctively felt that she would vote against the defendant. Despite not being selected as a juror, the spectator developed an interest in the case and had attended the entire trial. In an attempt to determine whether his instinct during jury selection was correct, the attorney approached the spectator and asked her whether she would have voted to convict the defendant. The spectator said, "I'd rather not talk about it." When the attorney explained that he was simply looking for constructive feedback, the spectator changed her mind and agreed to a brief interview. The attorney and spectator spoke for a few minutes, and the communication did not involve misrepresentation, coercion, duress, or harassment. Is the attorney subject to discipline? (A) Yes, because the spectator initially declined to speak with the attorney. (B) Yes, because post-trial contact with prospective jurors is prohibited. (C) No, because the communication did not involve misrepresentation, coercion, duress, or harassment. (D) No, because the spectator was not chosen for the jury.

A The attorney is subject to discipline because the spectator initially declined the attorney's request for an interview. ABA Model Rule 3.5(c) provides that after the trial is over and the jury is discharged, a lawyer must not communicate with a former juror or prospective juror if any of the following conditions is met: (i) local law or a court order prohibits such communication; (ii) the juror has told the lawyer that she does not want to communicate; or (iii) the communication involves misrepresentation, coercion, duress, or harassment. Here, the attorney violated the second condition—he persisted with his interview request after the spectator said that she did not want to talk with him. (C) is incorrect. Even though the communication did not involve coercion, duress, or harassment, the attorney still spoke with the spectator after she declined his request, violating the rule. (D) is incorrect because ABA Model Rule 3.5(c) applies to all jurors and even prospective jurors. (B) is too broad. There is no blanket prohibition regarding post-trial contact with jurors and prospective jurors. Rather, such communications are subject to conditions, and the attorney violated one of these conditions.

An insurance company offers a legal services insurance policy. In return for a yearly premium, an insured will be reimbursed by the insurance company for a specified amount for legal services during the year. The insured selects a lawyer from a list of "authorized providers" supplied by the insurance company. Any lawyer who agrees to follow a maximum fee schedule set by the insurance company can become an "authorized provider." The insurance company solicits insurance sales by in-person and live telephone contact with potential insurance buyers, working systematically through local telephone directories. Will an attorney be subject to discipline if he becomes an "authorized provider" and receives clients through the insurance company's insurance plan? (A) No, because the insurance company does not specifically target persons whom it knows are in need of legal services in a particular matter covered by its insurance plan. (B) No, because the insurance company's insureds are allowed to select whatever lawyer they wish from among the "authorized providers." (C) Yes, because the insurance company uses a specified maximum fee schedule. (D) Yes, because the insurance company uses in-person and live telephone solicitation to get business.

A The attorney will not be subject to discipline if he becomes an authorized provider under the insurance company's plan. The insurance company has set up a prepaid legal services plan of the kind referred to in the ABA Model Rules. A lawyer may receive legal business through such a plan, unless the operator of the plan uses in-person or live telephone contact to solicit people who it knows are in need of legal services in a particular matter covered by the plan. [ABA Model Rule 7.3(d)] (B) is wrong because there is no rule concerning the selection of counsel in a legal services insurance plan. (C) is also wrong because it would be an antitrust violation (and also an ethics violation) for a group of lawyers to conspire to follow a minimum or maximum fee schedule [see Arizona v. Maricopa County Medical Society, 457 U.S. 332 (1982)], but it is neither illegal nor unethical for a lawyer to agree to follow an insurance company's maximum fee schedule for work done for that company's insured. (D) is wrong because it is too broad. ABA Model Rule 7.3(d) permits in-person and live telephone solicitation by the operator of an insurance plan, except of persons who are known "to need legal services in a particular matter covered by the plan."

An attorney received her law degree two years ago from a small local college of law and technical sciences. Last summer she attended a three-day trial practice seminar at the Harvard Law School. During her brief career, she has tried five cases—two jury trials and three bench trials. She won both of the jury trials and two of the three bench trials. The attorney placed an ad under the subject heading "Trial Lawyers" in the classified pages of the local phone book. Her ad states in relevant part: "Trial Attorney Harvard Trained Never Lost a Jury Trial" Which of the following is correct? (A) To make the ad proper, the references to "Harvard Trained" and "Never Lost a Jury Trial" must be deleted. (B) To make the ad proper, the references to "Trial Attorney" and "Harvard Trained" must be deleted. (C) To make the ad proper, the references to "Trial Attorney" and "Never Lost a Jury Trial" must be deleted. (D) The ad is proper as written.

A The attorney would be subject to discipline for the last two statements in her advertisement. It is misleading for her to state that she is "Harvard Trained," because reasonable readers could interpret that to mean that she received her law degree from that school. [See ABA Model Rule 7.1 and comment 2] The statement "Never Lost a Jury Trial," although literally true, could create unjustified expectations and is therefore misleading. [See ABA Model Rule 7.1 and comment 3] The reference to "Trial Attorney" would not make the lawyer subject to discipline. Given her brief time in law practice, she has had significant experience as a trial lawyer. A lawyer is allowed to state the fields of law in which she does or does not practice. [ABA Model Rule 7.4(a)] (B), (C), and (D) are all incorrect because they would permit "Harvard Trained" or "Never Lost a Jury Trial" (or both) to remain in the advertisement.

In an effort to prevent homosexual persons from moving to their small town, a city council passed a zoning ordinance prohibiting the use of any dwelling house as a residence for two or more adults of the same sex unless they are related to each other. Violation of the ordinance is a misdemeanor and carries a fine of $10,000. A landlord who owns several rental houses in the city and does not want his potential renter pool limited hired an attorney to advise him. The attorney advised the landlord that the ordinance could probably be overturned under the state constitution, but that one would have to obtain legal standing to challenge the ordinance. The attorney advised that one way to obtain legal standing would be for a landlord to bring an appropriate action for declaratory judgment, and another way would be to simply violate the ordinance and raise the constitutional challenge as a defense to its enforcement. After receiving this advice, the landlord promptly rented one of his houses to two gay men. The landlord was cited for violation of the ordinance. The landlord subsequently brought an action to challenge the ordinance, and the ordinance was held unconstitutional. Was the attorney's conduct proper? (A) Yes, because violating the ordinance was one of the few ways to gain legal standing to challenge the constitutionality of the law. (B) Yes, because the ordinance was ultimately held unconstitutional. (C) No, because the attorney counseled and assisted the landlord in conduct the attorney knew was illegal. (D) No, because the attorney did not advise against renting houses to unrelated adults of the same sex.

A The attorney's conduct was proper because violating the ordinance was one of the ways to gain legal standing to challenge the constitutionality of the ordinance. A lawyer must not counsel or assist a client in conduct that the lawyer knows is criminal or fraudulent. However, a lawyer may counsel or assist a client to make a good faith effort to determine the validity, scope, meaning, or application of a law even if it requires the client to disobey the law. [ABA Model Rule 1.2(d)] This situation usually arises when a lawyer is asked how a particular law may be challenged, and the lawyer advises the client on ways to obtain legal standing, which include disobedience of the law. Here, the attorney was merely advising the landlord on methods of obtaining legal standing, including renting houses in violation of the ordinance.

A lawyer is admitted to practice only in one state. One of her regular clients is a corporation incorporated in the same state and has its principal place of business there. The president of the corporation went to another country to negotiate a contract that would be governed by foreign law. He telephoned the lawyer to ask whether a particular provision that he proposed would be lawful under foreign law, and he needed a quick answer because he had to resume negotiations in a few minutes. The lawyer had studied that country's law in school but warned the president about the danger of relying on off-the-cuff, unresearched legal advice. The president asked the lawyer to do the best she could. She then advised the president that the contract provision would be lawful. The president signed a contract that included the questioned provision. As it turned out, the provision violated that country's law and rendered the contract unenforceable. Is the lawyer subject to discipline? (A) No, because she did the best she could in an emergency situation. (B) No, because a lawyer in the United States is not expected to be competent in foreign law. (C) Yes, because she is not licensed to practice in another country (D) Yes, because she gave legal advice without adequate research.

A The lawyer is not subject to discipline because she did the best she could in an emergency situation. In an emergency situation, a lawyer may give legal advice on a matter that she would not be competent to handle in an ordinary situation. [Comment 3 to ABA Model Rule 1.1] Here, the lawyer's advice was limited to the narrow question the client posed. She is not subject to discipline because she did the best she could in the heat of the moment, and she warned the client about the dangers of relying on unresearched legal advice. (B) is wrong because a lawyer who renders advice about the law of another jurisdiction in an ordinary, nonemergency situation is expected to be competent to render such advice. [ABA Model Rule 1.1] (C) is wrong because the lawyer was not engaged in unauthorized practice when she advised her client about foreign law. She need not be admitted in another country in order to advise her local client about foreign law. (D) is wrong because the lawyer is not subject to discipline for rendering legal advice that turns out to be mistaken if she acted competently in light of the time-constrained circumstances in which she found herself.

When a lawyer was an associate at a law firm, she did the legal work for one of the firm's clients on a land sale transaction that earned the client millions of dollars. In gratitude, the client asked the lawyer whether she had any unfulfilled wishes. The lawyer told the client that she wished she had enough money to start her own solo law practice. The client then told her that he would lend her $100,000 to set up her new practice. In return, she would thereafter do all of his legal work at a 5% discount from her normal hourly fee, and she would pay off the $100,000 loan by monthly payments equal to 10% of her net income for the prior month. The lawyer was delighted. She drafted a complete, detailed agreement between herself and the client, and she advised the client in writing to obtain outside legal advice before signing the agreement. The client obtained the outside advice and signed the agreement, and the lawyer set up her solo practice accordingly. Is the lawyer subject to discipline? (A) No, unless she allows the client to interfere with her professional judgment in handling work for other clients. (B) No, unless the lawyer fails to give the law firm timely notice of the transaction. (C) Yes, unless the law firm consented to the loss of the client as a firm client. (D) Yes, unless the client is a lawyer.

A The lawyer is not subject to discipline unless she allows the client to interfere with her judgment in handling other clients' matters. [See ABA Model Rule 5.4] (B) is wrong because there is no law or disciplinary rule that requires the lawyer to notify the law firm regarding the transaction. (C) is wrong because no law or disciplinary rule requires the law firm's consent to the client's leaving the firm and giving his business to the lawyer as a solo practitioner. (D) is wrong because the loan payback clause does not violate the rule against splitting a legal fee with a nonlawyer. [ABA Model Rule 5.4(a)] True, the clause does measure the monthly payments as a percentage of the lawyer's net income in the prior month and most of her net income will probably come from legal fees. However, it makes sense to tailor her loan payments to her income, and the arrangement does not invite the evil that the no-splitting rule was designed to prevent—interference with the lawyer's professional judgment.

A lawyer is one of only nine lawyers who practice probate law in a particular county. In that county, all probate matters go before a single judge—the probate judge. The probate judge's duties include appointing counsel for the administrators of intestate estates. Serving as an administrator's counsel can be very lucrative. The incumbent probate judge recently retired. Her custom was to appoint out-of-county lawyers to serve as administrators' counsel, believing that such lawyers are less subject to local political and social pressures than county lawyers. The probate judgeship will be filled in six months in a partisan, contested election, and the lawyer is one of the candidates. The lawyer met jointly with the county's eight other probate lawyers and said he was seeking the probate judgeship, and would be making some important changes, such as appointing only local lawyers as counsel for administrators of intestate estates. He also said that if they'd like to contribute money or time, to please get in touch with his campaign committee via his website. Five of the eight lawyers sent generous monetary donations to the lawyer's campaign committee for the self-confessed purpose of securing legal appointments if the lawyer wins. The other three lawyers volunteered generous amounts of their time in the lawyer's campaign, but their motives for doing so were unexpressed and unclear. Which of the following is correct? (A) The lawyer is subject to discipline for personally soliciting money and publicly stated support from the eight lawyers. The five lawyers who contributed money will be subject to discipline if the lawyer wins and if they accept appointments from him. (B) The lawyer's conduct was proper because he was a candidate for a judicial office in a contested election. All eight lawyers, however, are subject to discipline because a practicing lawyer must not contribute either money or time to the political campaign of a judge before whom the lawyer expects to appear. (C) The lawyer is subject to discipline for promising to appoint only local lawyers as counsel for administrators, in the hope of securing the publicly stated support of the eight lawyers. The conduct of all eight lawyers, however, was proper because lawyers are permitted to support or oppose candidates in contested elections for judgeships. (D) The lawyer's conduct was proper because he was a lawyer-candidate, not a judge, at the time he met with the eight lawyers. The conduct of the five lawyers who contributed money was proper because a person's motive for making a political contribution is a private matter that is protected by the First and Fourteenth Amendments.

A The lawyer is subject to discipline under CJC Rule 4.1(A)(8), which prohibits a judicial candidate from personally soliciting campaign contributions or publicly stated support. The five lawyers who contributed money will be subject to discipline if the lawyer wins and if they accept appointments from him. [See ABA Model Rule 7.6] (B) is wrong because the lawyer is subject to discipline under CJC Rule 4.1(A)(8), as noted above. The second sentence of (B) is wrong because it overstates the constraints on a lawyer's participation in a judicial campaign. (C) is wrong because the five lawyers who contributed money violated ABA Model Rule 7.6, as noted above. The first sentence of (C) is a misapplication of CJC Rule 4.1(A)(13), which prohibits a judge from making pledges or promises that are inconsistent with the impartial performance of the judge's adjudicative duties "in connection with cases, controversies, or issues that are likely to come before the court." Here, the policy regarding whom the probate judge will appoint as administrators' counsel is not the kind of litigation issue to which the highlighted language of CJC Rule 4.1(A)(13), above, refers. The second sentence of (C) is wrong because the motive of the five money contributors was to obtain appointments if the lawyer won. [See ABA Model Rule 7.6] (D) is wrong because a lawyer who runs for a judicial post must follow the CJC. [See CJC Rule 4.1, comment 2] The second sentence of (D) is wrong because ABA Model Rule 7.6 trumps whatever privacy rights one can find in the peripheral glow of the First and Fourteenth Amendments.

A bank and trust company maintains a list of approved estate and trust lawyers as a service to their customers who seek advice on estate planning matters. When a young lawyer opened her trust and estate practice in town, she asked other lawyers how she could get on the bank's approved list. They explained that the bank lists lawyers who always name the bank in wills and trust agreements they draft for clients who need an institutional executor or trustee. The bank is one of the most stable and reputable banks in the state, and its fees for executor and trustee services are competitive with those of similar institutions. In light of what she has been told by the other lawyers, may the young lawyer seek to have her name included on the bank's list? (A) No, because a tacit condition of being on the list is always to name the bank as executor or trustee. (B) No, because a lawyer must not solicit business through an intermediary. (C) Yes, because naming the bank causes no harm to clients who need an institutional executor or trustee. (D) Yes, because those who use the bank's list are already bank customers.

A The young lawyer may not seek to have her name included on the list because naming the bank as executor or trustee in wills and trusts is a tacit condition of being on the list. A lawyer generally may not give anything of value to a person for recommending the lawyer's services. [ABA Model Rule 7.2(b)] When a lawyer names a bank as institutional executor or trustee for a client, the lawyer confers a monetary benefit on the bank in the form of the fees the bank will earn from the client's trust or estate. Here, the bank apparently lists only those lawyers who are willing to compensate it in this manner. That makes the arrangement "exclusive" and prevents the arrangement from being a reciprocal referral agreement of the kind permitted by ABA Model Rule 7.2(b)(4). Moreover, the bank's scheme creates a conflict of interest between the lawyer and the client who needs an institutional executor or trustee. The lawyer has a personal interest in staying on the bank's referral list, and that interest may skew the lawyer's judgment in advising a client whether to name the bank or a different institution. [See ABA Model Rule 1.7(a)] (B) is wrong because it is overbroad and does not hit on the specific problem with the bank's scheme. In proper circumstances, a lawyer may solicit business through an intermediary, e.g., through a prepaid legal service program or an approved lawyer referral service. (C) is wrong because a client is entitled to the unbiased advice of a lawyer in deciding what institutional trustee or executor to name. Even if the bank is just as good and its fees are as reasonable as other institutional fiduciaries, the bank's scheme deprives clients of unbiased advice. (D) is wrong because clients who use the bank for their routine banking needs might nevertheless desire to name some other institution as their executor or trustee; in any event, they are entitled to unbiased legal advice on that subject.

A lawyer is a partner in a private law firm. That firm regularly provides legal services to three major banks and two other important lending institutions in the community. The lawyer has been invited to become a member of the board of directors of the local legal aid society, the group that sets overall governing policies for the local legal aid office. One of the major issues that will soon face the board of directors is whether to amend the case intake guidelines to allow the legal aid office to represent clients in disputes with banks and other lending agencies. Which of the following statements is correct? (A) The lawyer may join the board of directors, but she must refrain from participating in the decision about the case intake guidelines. (B) The lawyer will be subject to discipline if she joins the board of directors because service on the board is in conflict with the interests of her firm's bank and lending institution clients. (C) It would be proper for the lawyer to join the board of directors, and it would be proper for her to participate in the decision about the case intake guidelines. (D) The lawyer may join the board of directors to help discharge her pro bono obligation, and she may vote in favor of amending the case intake guidelines in order to make it easier for low income persons to sue banks and other lending institutions.

A The lawyer may join the board of directors, but she must refrain from participating in the decision about the case intake guidelines. A lawyer may not participate in a legal service board decision that may adversely affect one of the lawyer's clients. [ABA Model Rule 6.3(b)] (B) is wrong because ABA Model Rule 6.3 encourages work with a legal services organization, even if the organization serves people whose interests conflict with the interests of the lawyer's clients. (C) and (D) are wrong because ABA Model Rule 6.3(b) prohibits a lawyer from taking part in a legal services organization decision if the decision will adversely affect one of the lawyer's clients.

A lawyer is on the in-house legal staff of a large corporation. In that capacity, she works daily with the corporation's top executive officers. She was assigned to defend the corporation in a lawsuit brought by a bank to collect a $750,000 promissory note. The note was signed on behalf of the corporation by its treasurer and chief financial officer. The corporation's defense is that the treasurer had no authority to sign the note and that the bank knew it. The corporation has advised the treasurer that it may seek indemnification from him if it is held liable to the bank. The treasurer is not represented by counsel. Shortly before the treasurer was to have his deposition taken by the bank, he called the lawyer and asked her what to expect at the deposition and how to respond to the bank's questions. What should the lawyer do? (A) Advise the treasurer to hire a lawyer to represent him at the deposition. (B) Tell the treasurer that she cannot discuss the matter with him unless he wants her to represent him at the deposition. (C) Advise the treasurer to tell the truth, to answer fully all questions that are asked, and to pause before each answer to give her time to object to the bank's questions. (D) Advise the treasurer that his own interests will be best served by answering truthfully and demonstrating, if he can, that he had authority to sign the note.

A The lawyer should not discuss the matter with the treasurer and should advise him to hire his own attorney. When an organization is the lawyer's client, the lawyer owes the duty of loyalty to the organization—not to the people who are its constituents. If there is a conflict between the interests of the organization and the interests of one of its constituents, the lawyer should advise the constituent to obtain independent legal counsel. [Comment 10 to ABA Model Rule 1.13] The lawyer's client is the corporation, and the corporation's interests are in conflict with the interests of the treasurer. If the corporation proves that the treasurer had no authority to sign the note, the bank may sue the treasurer himself. If the corporation is held liable to the bank, it may sue the treasurer for indemnification. In these circumstances, the treasurer needs his own lawyer, and the lawyer must not try to represent both him and the corporation. Furthermore, because the treasurer is presently unrepresented in the matter and his interests conflict with those of the corporation, the lawyer must not try to give him legal advice, except to get a lawyer. [See ABA Model Rule 4.3] (B) is wrong because the lawyer must not try to represent the treasurer at the deposition due to the conflict of interest explained above. (C) is wrong because the lawyer must not try to give the treasurer legal advice, except to get a lawyer. (D) is wrong for the reason just stated. Furthermore, as the corporation's attorney, the lawyer must not give legal advice to a person with conflicting interests (particularly advice to testify in such a way as to damage her client's position).

A law firm represented an oil company in a merger transaction in which the oil company acquired all of the assets of a smaller petroleum company in exchange for a specified amount of capital stock of the oil company. The law firm's work for the oil company was limited to the antitrust and securities law issues raised by the merger, and the firm lawyers who worked on the matter did not become privy to any confidential information concerning the routine operations of the oil company's business. The merger work was completed two years ago, and the law firm has not subsequently represented the oil company in any other matter. Recently, the law firm took in a new partner who had previously practiced as a solo practitioner. One of the cases that the new partner brought the firm from his solo practice was an employment discrimination case in which the new partner's client claims that the oil company fired him solely because of his age. When the new partner joined the law firm as a partner, the oil company promptly made a motion in the trial court to disqualify the new partner and the law firm as counsel for the plaintiff due to the law firm's earlier representation of the oil company in the merger matter. Are the new partner and the law firm subject to disqualification? (A) No, because the merger matter and the discrimination case are unrelated matters and because the law firm did not gain confidential information from the oil company that would be material in the discrimination case. (B) No, because whatever material confidential information the oil company might have picked up in the merger matter is not imputed to the new partner. (C) Yes, because the law firm owes continuing duties of loyalty and confidentiality to its former client, the oil company. (D) Yes, even though the merger matter and the discrimination case are unrelated and even though the law firm did not gain confidential information from the oil company that would be material in the discrimination case.

A The new partner and the law firm are not subject to disqualification because from the facts given in the question, it appears that the merger matter and the age discrimination case are not substantially related to one another, and that the firm lawyers did not gain confidential information in the merger matter that would be material to the discrimination case. [See ABA Model Rule 1.9] Therefore (C) and (D) are incorrect. (B) is incorrect because if the firm lawyers who worked on the merger matter had obtained confidential information that would be material in the discrimination case, their knowledge would be imputed to the new partner, even though he was not a member of the firm when the knowledge was obtained. [See ABA Model Rules 1.9, 1.10]

A probate attorney obtained a decedent's coin collection in order to inventory it. The attorney put the coin collection into a heavy brown envelope, labeled it as part of the decedent's estate, put the brown envelope and the decedent's other belongings into the file drawer of his desk, and left for lunch without locking the file drawer. The attorney's secretary saw the coins and saw what the probate attorney did with them. While the attorney was at lunch, the secretary took the envelope of coins and disappeared, never to be seen again. Is the probate attorney subject to discipline? (A) Yes, because the attorney did not put the coins in a safe place. (B) Yes, because the attorney is responsible for his employee's dishonest act. (C) No, because the loss was proximately caused by the secretary's dishonesty, not by the attorney's conduct. (D) No, because the attorney took reasonable precautions to safeguard the coins in the circumstances.

A The probate attorney is subject to discipline because he did not put the coins in a safe place. When a lawyer comes into possession of property to be held on a client's behalf, the lawyer must identify it as belonging to the client and must put it in a safe place. [ABA Model Rule 1.15(a)] Although the Rules do not define "safe," common sense suggests that allowing a valuable coin collection to be viewed by employees, placing it into an unlocked desk file, and then leaving the office is not safe. A lawyer should use the same level of care required of professional fiduciaries. The probate attorney's actions fall well short of that. (B) is wrong. The attorney may be liable to the estate in civil damages for his secretary's dishonest act, but the question here is professional discipline, not civil liability. The attorney could be disciplined if he did not take reasonable steps to train his secretary properly [ABA Model Rule 5.3], but if he took such steps, he should not be disciplined for her criminal act. (C) is wrong. The issue here is the attorney's failure to safeguard the coins; the proximate cause of the loss is beside the point. Technically, an attorney could be subject to discipline for failure to safeguard the property even if no loss occurred. (D) is wrong. Placing the coins in an unlocked desk file and leaving the office was not a reasonable way to safeguard them.

A client hired a lawyer to draft a will for him. The client willed his entire estate to a 43-year-old widow. The client told the lawyer in confidence that he was neither a relative nor a friend of the widow. The client explained that he felt a moral obligation to the widow because he had killed her husband, and he had never become a suspect or confessed his sin to anyone. One day after signing the will, the client committed suicide. In due course, all of the client's assets were distributed to the widow, and the probate court closed his estate and discharged his executor. The lawyer never told the widow or anyone else that the client had confessed to killing the widow's husband. Now, a few years later, an enthusiastic young prosecutor is charging an innocent man with murdering the widow's husband in the first degree with aggravating circumstances, and the prosecutor is seeking the death penalty. May the lawyer voluntarily tell the innocent man's defense counsel what his client told him in confidence about killing the widow's husband? (A) Yes, the lawyer not only may, but he must, tell the defense counsel what the client told him. (B) Yes, the lawyer may tell, but he would not be subject to discipline if he decides not to do so. (C) No, the lawyer would be subject to discipline if he told defense counsel because the attorney-client privilege survives the death of the client. (D) No, because the client's confidential confession to the lawyer would be inadmissible hearsay if offered against the prosecution in the murder trial.

B The controlling doctrine in this case is the lawyer's ethical duty of confidentiality, not the attorney-client privilege. The lawyer needs to know whether he can voluntarily reveal the client's confession, not whether he would be forced to do so if he were put on the witness stand in a court. ABA Model Rule 1.6(b)(1) states the applicable exception to the ethical duty of confidentiality: A lawyer may reveal confidential information if the lawyer reasonably believes that doing so is necessary to prevent reasonably certain death or substantial bodily harm. One might quibble whether the innocent man's death is "reasonably certain" when his trial has not even started, but surely the ethics rule should not be read to require the innocent man to order his last meal before being loosed from the executioner's grip. (A) is wrong because ABA Model Rule 1.6(b)(1) gives the lawyer discretion to reveal the client's confession; the Rule does not force him to do so. [See comment 15 to ABA Model Rule 1.6] (A few states go farther and require disclosure to prevent death or substantial bodily harm, but they are a small minority.) (C) is wrong for two reasons. First, the applicable doctrine is the ethical duty of confidentiality, not the attorney-client privilege. Second, even if the privilege were the applicable doctrine, who could claim it in this situation? The client cannot because he is dead. The client's executor cannot because the client's estate was closed and the executor was discharged. The lawyer cannot claim it because a lawyer's right to claim the privilege is only derivative from the client. (D) is wrong for two reasons. First, the admissibility of this hearsay is irrelevant to the ethics issue. Second, the client's confession would likely be admissible if offered by the innocent man against the prosecution because it is a declaration against penal interest by an unavailable declarant, and the client's will and suicide are independent evidence of the confession's trustworthiness. [See Fed. R. Evid. 804(b)(3); see also Chambers v. Mississippi, 410 U.S. 284 (1973)—due process violation where another man's confession was excluded in a murder trial]

For many years a lawyer has done business transactions work for a wealthy client. The client was recently injured in an automobile crash, and she has asked the lawyer to represent her as plaintiff in an action against the driver who injured her. The lawyer has taken some business cases to trial, but he has never handled a personal injury case. The lawyer would like to help his client and also generate some income. Which of the following would be an improper way for him to do so? (A) Take the case and, with the client's consent, associate a co-counsel who is competent in the field of personal injury law. (B) Refer the client to a competent personal injury lawyer and charge that lawyer a $1,000 forwarding fee. (C) Refer the client to a competent personal injury lawyer and charge the client a reasonable sum for the time spent in making the referral. (D) Take the case and, with the client's consent, undertake additional research to bring himself up to speed in the field of personal injury law.

B A "forwarding fee" is another term for a "referral fee," and payments for referrals are prohibited. [ABA Model Rule 7.2(b)] (A) is proper because a lawyer may take on a case that he is not competent to handle if he obtains his client's consent to associate a lawyer who is competent to handle it. [Comment 2 to ABA Model Rule 1.1] (C) is proper because a lawyer may refer her client to another lawyer who is competent to handle the case. Making a sound referral can take a significant amount of time, especially if the referring lawyer needs to research the backgrounds of several lawyers with whom she is not personally familiar. It is appropriate for the referring lawyer to charge her client for the time spent making the referral, subject of course to the general rule on reasonableness. [ABA Model Rule 1.5] (As a practical matter, however, many lawyers would not charge a regular client for making such a referral.) (D) is proper because a lawyer may take on a case that he is not competent to handle if he undertakes the study necessary to provide competent representation. [Comment 2 to ABA Model Rule 1.1]

A judge is a loyal member of the alumni association of the women's college from which she was graduated. The 25th reunion of her graduating class is coming up, and she has been asked to participate in some activities designed to raise money for a gift from the class to the college scholarship fund. Which of the following activities would be improper for the judge to do? (A) Make a substantial personal donation to the class gift fund. (B) Telephone other members of her graduating class and urge them to make a donation to the class gift fund. (C) Serve on the scholarship fund committee, which devises the various fund-raising strategies. (D) Attend a fund-raising dinner for the class gift.

B A judge may not personally solicit contributions for an organization other than from her family or certain other judges. [CJC Rule 3.7(A)(2)] Thus, (B) is improper. (A) is proper because Judge Jones, like anyone else, may contribute to any cause she likes. (C) is proper because a judge may assist an organization in planning fund-raising, although the judge may not actually participate in the fund-raising activity. [CJC Rule 3.7(A)(1)] A judge must not be a speaker or guest of honor at an organization's fund-raising event, but mere attendance at such an event is permissible. [CJC Rule 3.7(A)(4), comment 3] Thus, (D) is proper.

A lawyer who limits his practice to bankruptcy law has signed up on the local court roster of attorneys who are willing to take court-appointed criminal defense matters on a pro bono basis. He has taken approximately one such pro bono criminal case each of the past 10 years, but he has won only two of them. The day after tomorrow, the lawyer will start the jury trial of a criminal defendant charged with indecent exposure. This morning, the prosecutor held a press conference, at which he told reporters that this defendant had been accused of various sex offenses on six prior occasions. The prosecutor's statements are correct, but none of the prior incidents will be admissible in evidence at the upcoming trial. The defendant's lawyer thinks that the prosecutor was simply trying to poison the jury pool by degrading the defendant. The lawyer is planning to call his own press conference at which he will give the reporters the rest of the story. The lawyer intends to explain that on all six prior occasions, the defendant was arrested but never charged, and all six arrests were made by the same police officer, who holds a personal grudge against the defendant. Which of the following is correct? (A) The lawyer is subject to discipline for accepting this court appointment in light of his apparent lack of talent for criminal trial work. (B) The lawyer's proposed statements at the press conference are proper in light of the prosecutor's prior statements to the press. (C) The lawyer is subject to discipline for accepting court appointments in criminal matters when his active practice is limited to bankruptcy law. (D) The lawyer will be subject to discipline if he holds the press conference and makes the statements described above.

B A lawyer is permitted to "make a statement that a reasonable lawyer would believe is required to protect a client from the substantial undue prejudicial effect of recent publicity not initiated by the lawyer or the lawyer's client." [ABA Model Rule 3.6(c)] The lawyer is permitted to make these clarifying statements in response to the true, but incomplete, statements made by the prosecutor at a press conference. Thus, (D) is incorrect. (A) and (C) are incorrect because the lawyer has 10 years of experience handling pro bono criminal defense cases, and his win-loss ratio may be reflecting the underlying merits of the cases rather than the lawyer's trial ability.

A client hired an attorney to put together a complex real estate syndicate. In connection with that work, the client disclosed to the attorney a great deal of confidential information about the client's financial affairs. When the task was about half completed, the attorney's wife was killed in a car accident and his family's house burned down, all in the same week. The attorney was so emotionally and physically drained that he felt he could not competently continue with the work for his client. The client refused to allow the attorney to withdraw. The attorney begged the client to allow him to turn the files over to his law partner, an excellent real estate lawyer who was completely trustworthy and perfectly competent to handle the matter. The client refused to allow his files to be turned over to any other lawyer and insisted that the attorney himself promptly complete the work. What should the attorney do? (A) Turn the files over to his partner, and remain available to assist his partner to the extent possible. (B) Withdraw and turn the client's files over to the client. (C) Set the client's work aside until he recovers from the ills that have befallen him. (D) Continue with the matter and do the best that he can under the circumstances.

B A lawyer must withdraw if the lawyer's physical or mental condition will materially impair his ability to represent the client. [ABA Model Rule 1.16(a)(2)] The client may be right in thinking that hard work will be good for the attorney, but the attorney has to be the ultimate judge of his own physical and mental capacity to carry on. If the attorney believes that his condition prevents him from serving the client competently, he must withdraw regardless of what the client wants. (A) is wrong because the files include confidential information about the client's financial affairs, and the attorney cannot turn them over to his law partner against the client's express wishes. [ABA Model Rule 1.6] (C) is wrong because the client has asked the attorney to complete the work promptly. The attorney's recovery may take months or years. The attorney must not continue representing the client unless he can complete the work with reasonable diligence and promptness. [ABA Model Rule 1.3] (D) is wrong because, as discussed above, if the attorney believes his mental and physical conditions prevent him from serving the client competently, he must withdraw.

An attorney volunteers her legal services one night each week at a county legal advice hotline. The hotline is run under the auspices of the superior court, and it supplies free legal advice by telephone to callers who could not otherwise obtain legal services. Every caller assents to a "Statement of Understanding" at the outset of the call, informing the caller of the limited nature of the legal services that the hotline provides. Eighty-five lawyers volunteer their services as the attorney does. They come to the hotline office at various times on various days, and the volunteers hardly ever see or talk with each other. The nature of the hotline's work makes it impossible for the lawyers to conduct the kind of conflict-of-interest checks that an ordinary law firm would conduct before taking on a new client. One Tuesday night, the attorney counseled a distraught mother about her husband's physical and mental abuse of their school-age children. The attorney told the mother how to seek help from Child Protective Services. On the following Friday evening, a different hotline lawyer volunteer counseled the mother's husband about how to prevent Child Protective Services from scooping up his children and putting them in a foster home. Because the hotline does not do conflict-of-interest checks, the lawyer who counseled the husband had no way to know that the first attorney had counseled the mother a few nights before. Which of the following is correct? (A) The judges of the superior court are subject to discipline for permitting the hotline to operate under their auspices without making a conflict-of-interest check before dispensing legal advice to a new client. (B) Both lawyers and the judges of the superior court all acted properly because the hotline cannot do the kind of conflict-of-interest checking that a private law firm would do. (C) The first attorney acted properly in advising the mother, but the second lawyer is subject to discipline for giving legal advice to the father on the same subject. (D) Neither the mother nor the father was a "client" of the respective lawyers who advised them. Therefore, there was no conflict of interest, and both lawyers acted properly.

B The county legal advice hotline is the kind of operation envisioned in ABA Model Rule 6.5. Under ABA Model Rule 6.5, walk-in legal clinics, advice-only clinics, legal advice hotlines, and the like are not held to the high conflict-of-interest standards that govern ordinary law offices. Legal hotlines, walk-in clinics, and similar providers of quick legal service typically operate under conditions that make it difficult or impossible to conduct ordinary conflict-of-interest checks. Under ABA Model Rule 6.5(b), the second lawyer would be subject to discipline only if he actually knew that the first attorney had previously counseled the mother of the abused children. A lawyer's actual knowledge can be inferred from the circumstances [ABA Model Rule 1.0(f)], but the question does not mention any circumstances from which an inference could be made that the second lawyer had actual knowledge. (A) is wrong because it fails to account for ABA Model Rule 6.5. (C) is wrong because, absent actual knowledge of a conflict, the rule of imputed disqualification does not apply between two lawyers in a quick-legal-service program. [See comment 4 to ABA Model Rule 6.5] (D) is wrong because both the mother and father were "clients" of the respective lawyers who advised them. [Comment 1 to ABA Model Rule 6.5] This is important because ABA Model Rule 6.5 loosens only the conflict-of-interest rules, not other aspects of the lawyer-client relationship, such as the duty of competence, the duty of diligence, and the duty of confidentiality.

A technology company makes computer chips. It is incorporated in State A and it has a chip manufacturing plant in State B. Recently, the chip manufacturing plant has been afflicted with a rash of employment discrimination claims. The company's general counsel instructs one of the company's lawyers, who is admitted to practice only in State A, to move temporarily to State B, settle all existing meritorious claims, prepare all nonmeritorious claims for trial, and train the managers of the State B plant to comply with federal and state employment discrimination laws. The general counsel knows that the lawyer is not admitted to practice in State B. State B requires out-of-state lawyers to seek pro hac vice admission before undertaking pretrial preparation. May the lawyer do as the general counsel has instructed? (A) Yes, because the lawyer's right to practice temporarily in State B is a debatable issue of legal ethics, and it is appropriate for such an issue to be decided by a lawyer's supervisor. (B) Yes, but because State B requires out-of-state lawyers to be admitted pro hac vice in order to engage in pretrial preparations, the lawyer must seek such admission. (C) No, because the lawyer must take and pass State B's bar exam. (D) No, because the lawyer must associate a State B lawyer who will actively participate with the lawyer in settling the meritorious claims and preparing the nonmeritorious claims for trial.

B ABA Model Rule 5.5(c) concerns temporary practice in a state where the lawyer is not admitted. Rule 5.5(c)(4) permits the lawyer to practice temporarily in State B to the extent that his work in that state is reasonably related to the work he does for the company in State A. However, because State B requires out-of-state lawyers to seek pro hac vice admission before engaging in pretrial preparation, ABA Model Rule 5.5(c)(2) and (3) require the lawyer to seek such admission.

A personal injury lawyer and an orthopedic surgeon are good friends, and they have a high mutual regard for each other's professional abilities. One day on the golf course, they made a reciprocal referral agreement: whenever the lawyer has a personal injury client with need for an orthopedic surgeon, the lawyer promised to refer the client to the surgeon. Similarly, whenever the surgeon has an injured patient with a need for a personal injury lawyer, the surgeon promised to refer the patient to the lawyer. The agreement was oral, not written, and there was no mention of an expiration date; both women simply assumed that the agreement would continue indefinitely until one or the other wanted to end it. Likewise, they did not discuss whether the agreement would be exclusive; both women simply assumed that neither of them would refer someone to a competitor of the other. Was it proper for the lawyer to make this agreement with the surgeon? (A) No, because the agreement was not reduced to writing. (B) No, because the agreement was of an indefinite duration. (C) No, because a lawyer must not give anything of value to a person for recommending her services. (D) No, because a lawyer must not enter into a reciprocal referral agreement with a nonlawyer.

B ABA Model Rule 7.2(b)(4) permits a lawyer to make a reciprocal referral agreement with another lawyer, or with a nonlawyer professional, if the agreement is not exclusive and the referred person is told about the agreement. However, comment 8 to ABA Model Rule 7.2 cautions lawyers that such an agreement should not be indefinite in duration. (A) is wrong because ABA Model Rule 7.2(b)(4) does not require a reciprocal referral agreement to be in writing. (C) is wrong because reciprocal referral agreements are one of four exceptions to the general rule that a lawyer must not give something of value for a referral. [See ABA Model Rule 7.2(b)] (D) is wrong because ABA Model Rule 7.2(b)(4) expressly permits reciprocal referral agreements with nonlawyer professionals.

For three years, an attorney was a partner in a law firm. During that period, the attorney represented a client, Alpha, in obtaining a business loan from a bank. Alpha disclosed to the attorney a great deal of confidential information about his business and his personal assets. No other attorney in the law firm gained access to that confidential information. Recently, the attorney died. Shortly thereafter, Beta asked the senior partner of the law firm to represent him in a civil suit for serious personal injuries Beta suffered when he was run over by a delivery truck driven by one of Alpha's employees. Would it be proper for the senior partner to represent Beta? (A) Yes, because the information obtained by the deceased attorney about Alpha's assets has no effect on liability in a personal injury suit. (B) Yes, because neither the senior partner nor any other attorney in the law firm gained access to Alpha's confidential information. (C) No, because the conflict created by the deceased attorney's work for Alpha is imputed to the senior partner. (D) No, because the senior partner did not obtain Alpha's informed consent, confirmed in writing.

B It would be proper for the senior partner to represent Beta because neither the partner nor any other lawyer in the firm gained access to Alpha's confidential information. Even though Alpha's bank loan is not substantially related to Beta's personal injury suit, the confidential information that the deceased attorney got from Alpha may well become important in Beta's suit. Knowing the extent and nature of Alpha's assets could be of great value to counsel for Beta in advising Beta whether to settle or in collecting on a judgment against Alpha. Here, however, the deceased attorney was the only attorney who gained access to Alpha's financial information, and the attorney is now dead. Under ABA Model Rule 1.10(b), the senior partner may represent Beta, because neither he nor any other attorney remaining in the firm had access to Alpha's confidential information. (A) is wrong because it ignores the issue presented by the confidential information. (C) is wrong because ABA Model Rule 1.10(b) creates an exception to the ordinary rule that confidential information gained by one lawyer in a firm is deemed to be known by all lawyers in the firm. (D) is wrong because ABA Model Rule 1.10(b) allows the senior partner to serve even without the consent of Alpha.

A farmer asked an attorney to represent him in an eminent domain proceeding in which the state sought to obtain a right-of-way across the farmer's land. The attorney had not handled an eminent domain case before, but she planned to make herself competent through diligent research and study. As it turned out, the attorney did not have enough time to do what she had planned, so she associated an eminent domain specialist as her co-counsel in the case. The attorney did not consult the farmer about associating the specialist. The specialist did about 90% of the work in the case, and the attorney did the other 10%. Together they secured a very favorable result for the farmer, and the attorney sent the farmer a fee bill for a reasonable amount. The farmer paid the bill, and the attorney remitted 90% of the proceeds to the specialist. Is the attorney subject to discipline? (A) Yes, because she took on a case that she was not competent to handle. (B) Yes, because she did not consult the farmer about associating the specialist. (C) No, because the fee split was in proportion to the work done by the two lawyers. (D) No, because she associated a co-counsel who was competent to handle the case.

B The attorney is subject to discipline because she did not consult the farmer about associating the specialist. A lawyer may split her fee with a lawyer outside her firm if the total fee is reasonable, the split is either in proportion to the work done (or in some other proportion if the splitting lawyers assume joint responsibility), and the client agrees to the split in a writing that discloses the share that each lawyer will receive. [ABA Model Rule 1.5(e)] Because the attorney did not tell the farmer about the arrangement, she is subject to discipline. (A) is wrong because a lawyer may take on a case she knows she is not competent to handle if she prepares as needed to become competent, or if she associates a lawyer who is competent to handle it. [Comment 1 to ABA Model Rule 1.1] (C) and (D) are wrong because they ignore the need to inform the farmer.

A judge sits on a federal appellate court. He and two other federal judges heard a diversity of citizenship case in which they were required to interpret a state statute concerning the marital communications privilege. The judge's two colleagues wrote the majority opinion, in which they concluded that the statute gives only the witness-spouse the right to claim the privilege. The judge wrote a vigorous and scholarly dissent, arguing that the statute gives both spouses the right to claim the privilege. Later, a state senator introduced a bill to amend the statute to reflect the judge's position. The state senate invited the judge to testify about the public policy reasons for giving both spouses the right to claim the privilege. May the judge testify? (A) Yes, but only if the two judges who wrote the majority opinion are also allowed to testify. (B) Yes, because a judge may engage in activities designed to improve the law. (C) No, because a judge must not become involved in politics, subject to certain exceptions that do not apply here. (D) No, because a judge is not allowed to make public statements about disputed propositions of law, except when acting in his judicial capacity.

B The judge may testify at a public hearing in connection with matters concerning the law. [CJC Rule 3.2(A)] (A) is wrong because there is no rule requiring "equal time." (C) is wrong because it is overbroad. The general rule against judicial involvement in politics limits only some types of political activities, not including legislative testimony. [CJC Canon 4] (D) is wrong because, with respect to issues that are likely to come before the court, a judge is prohibited from making pledges, promises, or commitments that are inconsistent with the impartial performance of his duties. [CJC Rule 4.1(A)(13)] That Rule does not apply here because the judge's testimony, which would be designed to improve the law, would not constitute a promise that is inconsistent with the performance of his adjudicative duties.

The mother of a full-time trial judge owns a small business that she wishes to sell. After she and a prospective buyer come to terms on the sale, the buyer has his lawyer draw up a sales contract and presents it to the judge's mother, who asks her son, the judge, to review it for her. The judge agrees, marks up the contract, and returns it to his mother to present to the buyer's attorney. The judge's mother did not tell the buyer that her son reviewed the contract. The buyer has no known reason for being likely to appear in the judge's court. Were the judge's actions proper? (A) Yes, because the buyer is not likely to appear in the judge's court in the future. (B) Yes, because he did not charge his mother a fee. (C) No, because a full-time judge is not permitted to practice law. (D) No, because the judge's identity was not disclosed to the buyer or the buyer's attorney.

B The judge's actions were proper because he did not charge his mother a fee. Although a full-time judge may not practice law, there is an exception for this type of transaction. A judge may, without compensation, give legal advice to, and draft and review documents for, a member of the judge's family. [CJC Rule 3.10] (A) is wrong because whether the buyer might appear in the judge's court does not affect the judge's ability to prepare documents for his mother. If the buyer does appear in his court, the judge's participation in that proceeding will be evaluated at that time in light of the facts. (C) is wrong because, as stated above, there is an exception for reviewing documents for relatives without compensation. (D) is wrong because, although the judge cannot act as a negotiator, there is no requirement that his identity as the person who made the revisions be kept a secret.

A justice was on the state supreme court. The state's supreme court rules provide that in capital punishment cases, any one justice of the supreme court is empowered to grant a stay of execution pending appeal to the supreme court. The justice granted such a stay in a recent criminal case, on the ground that the defendant had been denied the effective assistance of counsel at his trial. A few months later, the justice retired from the supreme court and went back to private law practice. In due course, the supreme court heard the appeal in the case, rejected the defendant's effective assistance of counsel contention, and affirmed the death penalty. The defendant then commenced a federal habeas corpus proceeding in an appropriate federal district court and asked that court to appoint a private lawyer to represent him. The district court appointed the retired justice to represent the defendant. A key issue in the habeas corpus proceeding is whether the defendant was deprived of the effective assistance of counsel at his trial. May the retired justice represent the defendant without getting informed consent, confirmed in writing, from all parties to the habeas corpus proceeding? (A) No, because there is reasonable ground to doubt the justice's impartiality in the matter. (B) No, because when the retired justice was a supreme court justice he granted a stay of execution to the defendant. (C) Yes, because the retired justice was appointed by the district court, and his prior involvement in the matter is not sufficient grounds for refusing the appointment. (D) Yes, because the respondent in the habeas corpus case is the prison warden, not the state.

B The justice may not represent the defendant in the habeas proceeding because, while serving as a supreme court justice, the retired justice granted the defendant a stay of execution. A lawyer must not represent a client in a "matter" in which the lawyer earlier participated "personally and substantially" as a judge. [ABA Model Rule 1.12(a)] The habeas corpus proceeding and the earlier appeal should be regarded as the same matter because the habeas corpus proceeding will doubtless raise many of the same issues that were decided on the earlier appeal. (The effective assistance of counsel issue is one example.) The stay of execution should be regarded as personal, substantial participation. (A) is wrong because it confuses the roles of judge and lawyer; a judge is expected to be impartial, but a lawyer is expected to be a partisan. (C) is wrong because one of the proper reasons for turning down a court appointment is that it would require the lawyer to violate a disciplinary rule. [ABA Model Rule 6.2(a)] (D) is wrong because it elevates form over substance; the similarity of legal issues, not the case caption, should be determinative here.

An entertainment lawyer has for many years represented a country music star. One evening, the lawyer and the music star were having a quiet business dinner together at a restaurant. Another diner approached their table and in a loud voice began a vulgar and defamatory tirade against the music star. Everyone in the restaurant heard the entire exchange. While all of the defamatory comments about the music star involved her personal life, about which the lawyer had no real knowledge, he felt that they could not possibly be true. At the music star's request, the lawyer commenced a slander suit against the diner, after spending considerable time reacquainting himself with slander law. In his answer to the complaint, the diner admitted making the allegedly slanderous statements, and as an affirmative defense, he alleged that the statements were entirely truthful. When the case comes to trial, would it be proper for the lawyer to act as the music star's trial counsel? (A) Yes, if the star gives informed consent, confirmed in writing. (B) Yes, because the lawyer is not a necessary witness. (C) No, because there is a possibility that the lawyer may be called as a witness. (D) No, because he was not competent to take a slander case.

B The lawyer may act as the music star's trial counsel because he is not a necessary witness. [ABA Model Rule 3.7(a)] A roomful of witnesses heard the diner's comments and could testify to them. Moreover, the diner has made a judicial admission that he made the statements; thus, no testimony is required on that point. The lawyer has no knowledge as to the truth of the statements, as he knows nothing of the star's personal life; thus, he would have no relevant testimony on that issue. Therefore, the lawyer is neither a "necessary" witness, nor a witness who "ought" to be called. (A) is wrong because there is no need for informed consent in this situation. (C) is wrong because even if there is a remote possibility that the lawyer might be called, he is not a necessary witness, and it is unlikely that he would be called by the diner's lawyer because he could not have anything favorable or relevant to add. A mere remote possibility that the lawyer will be called as a witness is not sufficient to disqualify the lawyer from representing a client. (D) is wrong because every attorney generally is considered competent to take any case as long as the attorney sufficiently prepares. Here, the attorney spent considerable time preparing before he filed the complaint.

A lawyer is defending a marine supply company in a civil action brought by the state attorney general under a statute that makes it a civil offense for any person or business entity to bribe or give a kickback to a state official. The statute authorizes fines of up to $100,000 per transaction for any violation. The marine supply company has a strict corporate policy that prohibits its employees from bribing or giving kickbacks to anyone. Employees who violate the policy are subject to immediate discharge and are required to indemnify the marine supply company for any loss it suffers as a consequence of the violation. The attorney general has noticed the depositions of dozens of the marine supply company's employees. One of these employees, prior to his recent retirement, was the sales manager of the marine supply company. The lawyer met with this employee to prepare him for his deposition. At the outset of the interview, the lawyer agreed to represent the employee without charge, and the lawyer told the employee that anything said between them would be confidential. During the interview, the lawyer asked the employee whether he had ever bribed any state officials. The employee confessed that he had, but said it had been necessary because all of the company's competitors were doing it, too. What course of action may the lawyer pursue at this point? (A) Withdraw from the case and inform the attorney general what the employee said. (B) Withdraw from the case and keep the employee's statement in confidence. (C) Withdraw from representing the employee and inform the marine supply company what the employee said. (D) Continue in the case, inform the marine supply company what the employee said, and advise the marine supply company to seek prompt settlement.

B The lawyer may withdraw from the case and keep the employee's statement in confidence. When an organization is the lawyer's client, the lawyer owes a duty of loyalty to the organization. When the interests of the organization and its constituents conflict, the lawyer should remind the person that the lawyer represents the organization and not the person. It would be appropriate for the lawyer to remind the person that communications between them may not be protected by the attorney-client privilege, and that the person may want to obtain independent counsel. [ABA Model Rule 1.13] Here, the lawyer should not have asked the question unless he was prepared for an affirmative answer. He should have known before asking that if the answer was yes, the employee's interests and the marine supply company's interests would conflict; thus, the lawyer should not have offered to represent the employee, and certainly should not have promised to keep the employee's statements in confidence. (Note that the lawyer could be subject to discipline for this conduct.) The issue here, however, is what course of action the lawyer may now take. Now that the lawyer has agreed to represent the employee, and the employee has confessed in confidence, the only thing the lawyer can do is withdraw from the matter entirely and keep the employee's confession in confidence. [See ABA Model Rule 1.9] (A) is wrong because he would violate the employee's confidence by disclosing the confession to the Attorney General. (C) and (D) are wrong because revealing the employee's confession to the marine supply company would also violate the employee's confidence.

A lawyer regularly represents a manufacturer of electric kitchen appliances. One morning the president of the manufacturing company called the lawyer and asked if the lawyer had seen the newspaper story about a woman who was electrocuted when she opened the door of her dishwasher. The company president stated that he believed the dishwasher was one that his company had manufactured. The company president also stated that he found some quality control records from that period which reflected that some dishwashers left the plant without proper testing. He continued that the records should have been shredded, but somehow had been overlooked, and said that he intended to send the records to the shredder immediately unless the lawyer told him that he could not. Must the lawyer advise the president to keep the records? (A) Yes, unless the company has a clearly established policy of shredding quality control records after two years (B) Yes, because the records have potential evidentiary value if the company gets sued. (C) No, because at this point there is no litigation pending against the company respecting this matter. (D) No, unless it was certain that the company was the manufacturer of the dishwasher in question.

B The lawyer must advise the president to keep the records because the records have potential evidentiary value if the company is sued. A lawyer must not counsel or assist a person to destroy material that has "potential evidentiary value." [ABA Model Rule 3.4(a)] Although it is not certain that the company manufactured the dishwasher in question, the president said he was "pretty sure that it was one of ours." Furthermore, it is not certain that the company will be sued if it was one of their washers, but the chances are good that it will be. If commencement of proceedings can be foreseen, the documents have potential evidentiary value and cannot be destroyed. [Comment 2 to ABA Model Rule 3.4] (A) is wrong because even if the company's records retention program called for the routine shredding of these records long ago, they were not shredded then, and they have potential evidentiary value now. (C) is wrong because commencement of proceedings is foreseeable. (D) is wrong because the standard is backward. The records should be preserved until the company is certain that the dishwasher in question was not manufactured by the company.

After a series of brutal muggings, the police captured a person whom they charged with the crimes and the person was eventually ordered to stand trial. Two days before jury selection for the case began, a local newspaper reporter cornered the prosecutor, a district attorney, in a cafe. The district attorney said she was certain the defendant was the right man because, among other things, they had discovered that he was previously convicted three times for brutal muggings in other states. Is the district attorney subject to discipline for making the statement to the reporter? (A) No, because prior criminal convictions are a matter of public record. (B) No, because a lawyer has a First Amendment right to inform the public about pending cases. (C) Yes, because the district attorney should have known that the statement would be quite likely to prejudice the trial. (D) Yes, because a prosecutor must not make public comment on a pending case.

C The district attorney is subject to discipline because she should have known that the statement was likely to prejudice the trial. A lawyer who is connected with a case must not make a public statement outside the courtroom that the lawyer reasonably should know would have a substantial likelihood of materially prejudicing the case. [ABA Model Rule 3.6(a)] The district attorney's comment to the reporter falls in that category because it revealed very damaging material that had not been, and probably would not be, admitted into evidence. A prosecutor must not make extrajudicial comments that have a substantial likelihood of heightening public condemnation of the accused. [ABA Model Rule 3.8(f)] The district attorney's statement also runs afoul of this rule by revealing prior crimes that would heighten public condemnation of the defendant. (A) is wrong because even if the prior convictions are a matter of public record, that does not absolve the district attorney for making a statement she knew would substantially prejudice the case. Furthermore, while in a technical sense the defendant's prior convictions in other states are matters of public record (if one knew where to go and what to look for, one could dredge them out of the court records of the other states), the prior convictions were doubtless not part of the public record in the defendant's present prosecution. (B) is wrong because it is too broad; lawyers do have First Amendment rights to express themselves about pending cases, but those rights are limited by the due process rights of litigants to fair court proceedings. (D) is wrong because it is also too broad; a prosecutor may make some kinds of public comment about pending cases, but not comments that are likely to cause prejudice.

Attorneys Alpha and Beta have been law partners for six years. Beta was suspended from practice for one year based on an intentional tax law violation. Alpha took over Beta's clients when his suspension went into effect. Shortly before his suspension, Beta had negotiated a $30,000 personal injury settlement with an insurance company on behalf of his client. Two weeks after the settlement was reached, the insurance company sent a $30,000 check to the law offices. By this time Beta's suspension had gone into effect. Alpha placed the check in the proper firm account and confirmed the amount of the fee with Beta's client. Alpha then promptly forwarded a $20,000 check to the client and a $10,000 check to Beta, the latter check representing Beta's one-third contingent fee. Is Alpha subject to discipline? (A) Yes, because she should have held the $10,000 in the client trust account until Beta's suspension had ended. (B) Yes, because a lawyer is prohibited from sharing legal fees with a nonlawyer. (C) No, because Beta earned the fee prior to his suspension. (D) No, because the $10,000 belonged to Alpha's firm, and she could do anything she wished with it, including sending it to Beta as a gift.

C Alpha is not subject to discipline because Beta earned the fee prior to his suspension. Despite his suspension, Beta is entitled to the fees he earned while he was still lawfully practicing law. It is true that a lawyer is prohibited from aiding a nonlawyer in the unauthorized practice of law, but here Beta is not practicing law, and Alpha is merely forwarding his previously earned fee. (A) is wrong because Beta need not wait until he is reinstated to collect a fee he earned prior to his suspension. (B) is wrong because Beta is not a nonlawyer, and Alpha is not splitting legal fees with him in any case. Alpha is merely transmitting Beta's own money to him. (D) is wrong because the money does not belong to Alpha, and even if it did, she cannot do anything she wishes with it. For example, there are rules prohibiting the sharing of legal fees with nonlawyers.

An attorney recently opened his solo law practice in a small town. His practice is fairly evenly divided between civil litigation and criminal defense. The Superior Court has just appointed the attorney to represent two defendants, who will be tried jointly for their alleged kidnapping and brutal murder of nine local school children. Which of the following is not a valid reason for the attorney to decline the appointment? (A) He believes that to represent the two defendants will take so much time away from his newly opened practice as to impose an unreasonable financial burden on him. (B) He believes that one defendant coerced the other defendant into helping kidnap and kill the children. (C) He believes many of his potential clients will be outraged if he represents the two defendants. (D) He believes that confidential information he received when representing one of the prosecution's key witnesses will be useful in impeaching that witness's credibility.

C A lawyer can be disciplined for trying to avoid a court appointment without good cause. [ABA Model Rule 6.2] The reason stated in (C) is not an acceptable reason for declining the appointment; a lawyer has a duty to represent his fair share of indigent or unpopular clients. [Comment 1 to ABA Model Rule 6.2] (A) is a legitimate reason for declining an appointment. A lawyer is permitted to turn down a court appointment if it "is likely to result in an unreasonable financial burden." [ABA Model Rule 6.2(b)] (B) is also a legitimate basis for declining appointment because a lawyer may turn down a court appointment if it is likely to cause the lawyer to violate a rule of professional conduct. [ABA Model Rule 6.2(a)] If the two defendants are to be tried jointly, and if one defendant did coerce the other defendant into helping with the kidnapping and killing, there is a sharp conflict of interest between the two defendants. [See ABA Model Rule 1.7(a)] It would be an ethical violation to represent co-defendants with conflicting interests (consent will not solve the conflict); thus, the attorney can decline the appointment on this ground. (D) raises another conflict of interest that would justify the attorney in declining the appointment. Had he not gained confidential information from the prosecution's witness, he might have discovered that information independently and been able to use it to impeach the witness. As it stands, however, his ability to impeach is constrained by his duty not to use the confidential information to the disadvantage of the witness, his former client. [ABA Model Rule 1.9(c)(1)]

A lawyer takes on a client who was hit by a car and was unable to work for 18 months. At the client's request, the lawyer agreed to represent the client on a contingent fee basis. The client requested that the lawyer provide him with certain financial assistance during the pendency of the lawsuit. The lawyer agrees to provide a $20,000 loan, in an agreement reviewed by independent counsel. The $20,000 would cover $5,000 to support the client's family, $5,000 for the client's medical treatment, $5,000 to retain an expert witness, and $5,000 for job training. Which of the following was proper? (A) The $5,000 to support the client's family during the pendency of the suit. (B) The $5,000 for the client's medical treatment. (C) The $5,000 to pay the expert witness fee. (D) The $5,000 for job training.

C A lawyer may advance court costs and litigation expenses on the client's behalf. A lawyer is subject to discipline, however, for rendering any other kind of financial assistance to a client in the context of contemplated or pending litigation. [ABA Model Rule 1.8(e)] (C) represents court costs and litigation expenses, which are permissible. Family support is not an expense of litigation; thus (A) is improper. Similarly, treatment of the client's injuries and job retraining are not expenses of litigation, therefore (B) and (D) also are improper.

An attorney in solo practice published a brochure regarding what one should do when injured. The brochure contains accurate, helpful information about obtaining proper medical treatment, recording details of the accident, notifying insurance companies, not making harmful statements, and the like. The attorney's name, address, and telephone number are printed on the brochure's cover. One afternoon, the attorney saw a pedestrian knocked down in a crosswalk by a hit-and-run driver. He and another bystander called 911 and gave the pedestrian emergency first aid until an ambulance arrived. The next day, the attorney visited the pedestrian in the hospital and gave the pedestrian a copy of his brochure. Which of the following is correct? (A) The attorney is subject to discipline, both for publishing the brochure and for giving the brochure to the pedestrian in the hospital (B) The attorney is subject to discipline for publishing the brochure. (C) The attorney is subject to discipline for giving the pedestrian a copy of the brochure at the hospital. (D) The attorney's conduct was proper because the brochure's contents are neither false nor misleading.

C A potential problem arises when the content of a brochure is improper or when the printed material is used as part of an act of solicitation. Here, the content was proper, but when the attorney gave the brochure to the hospitalized pedestrian, he crossed the line into impermissible in-person solicitation. [See ABA Model Rule 7.3; see also Ohralik v. Ohio State Bar Association, 436 U.S. 447 (1978)—noting the particular potential for undue influence and overreaching in the hospital context, where individuals are often especially vulnerable] (A) and (B) are incorrect because publishing the brochure, in and of itself, is not problematic, just as the act of printing business cards is fully permissible. (D) is incorrect because the truthfulness of the brochure's contents does not counteract the attorney's impropriety in providing his contact information during a hospital visit.

The state bar certifies specialists in nine fields of law, one of which is tax law. An attorney has not yet earned her certificate of specialization in tax law, but she is working toward that goal. The attorney's ad states that she is a specialist in tax law, and that tax law is a field in which the state bar grants certificates of specialization. The attorney limits her practice to tax matters; she refers all other kinds of legal matters to a solo practitioner in general practice. The solo practitioner, in turn, refers all tax matters to the attorney. Without exception, the solo practitioner and the attorney have followed that pattern of referrals for five or six years; they have no formal reciprocal referral agreement, but each invariably follows the pattern, expecting the other to reciprocate. Which of the following most correctly describes the attorney's situation? (A) The attorney is subject to discipline for her ad, but not for maintaining the referral relationship with the solo practitioner. (B) The attorney's ad is proper, and so is her referral relationship with the solo practitioner. (C) The attorney is subject to discipline for her ad, and her referral relationship with the solo practitioner is improper because it would need to be nonexclusive, and the two lawyers would need to disclose it to referred clients. (D) The attorney's ad is proper, but she is subject to discipline for maintaining the referral relationship with the solo practitioner.

C ABA Model Rule 7.4(d) prohibits a lawyer from stating or implying that she is a certified specialist unless she has been certified by an appropriate organization that is clearly identified in the lawyer's communication. The attorney's ad appears to have been artfully crafted to make unsophisticated readers think that the attorney has been certified by the state bar. Thus, the ad violates both ABA Model Rule 7.4(d) and ABA Model Rule 7.1, which prohibits misleading advertising. As for the attorney's reciprocal referral relationship with the solo practitioner, the applicable rule is ABA Model Rule 7.2(b)(4), which permits a reciprocal referral agreement between lawyers, provided that the agreement is nonexclusive and the referred clients are told about the existence and nature of the agreement. ABA Model Rule 7.2(b)(4) has not yet been prominently interpreted, leaving one to wonder whether the relationship between the attorney and the solo practitioner should be regarded as an "agreement." On the one hand, the question states that they have no "formal reciprocal referral agreement." On the other hand, in some legal contexts, a consciously reciprocal course of dealing can be the equivalent of an agreement. [See, e.g., United States v. Container Corp. of America, 393 U.S. 333 (1969)—competitors' reciprocal exchange of price data was held to be an agreement under section 1 of the Sherman Antitrust Act] However, for purposes of this question, the relationship between the attorney and the solo practitioner will be deemed an agreement under ABA Model Rule 7.2(b)(4) because it is bound to influence their judgment about referrals to some degree. Here, the relationship violates the nonexclusive requirement because the two lawyers follow the pattern "without exception." Furthermore, the relationship would be proper only if the attorney tells referred clients about the relationship so that they can decide for themselves how to value the referral. One can reach the same conclusion by applying ABA Model Rule 1.7(a)(2)—the attorney's reciprocal relationship with the solo practitioner gives the attorney a personal interest (obtaining future referrals) that is in conflict with the interest of her client (obtaining an unbiased referral). The conflict could be solved only by full disclosure and informed consent of the affected client, confirmed in writing. [ABA Model Rule 1.7(b)]

Solo practitioners Alpha and Beta share office space. Each of them has organized her practice as a professional corporation. Alpha and Beta frequently consult each other about their respective cases, and they often refer clients to one another. Sometimes they work on cases together under a fee-sharing arrangement. When one of them is out of the office, the other responds to client inquiries to the extent that she is able, and to facilitate that practice, each attorney has physical access to the other's client files. A plaintiff hired Alpha to sue a bakery for personal injuries he sustained when he bit into a piece of glass in a dinner roll baked by the bakery. The bakery's liability insurance carrier asked Beta to serve as defense counsel in the case. Alpha and Beta each disclosed her relationship with the other to their clients, and the plaintiff, the bakery, and the insurance company each gave written consent to Beta's serving as defense counsel. May Beta take the case? (A) Yes, because Alpha and Beta believe that they can effectively represent their respective clients. (B) Yes, because the rule of imputed disqualification does not apply to Alpha and Beta. (C) No, even though Alpha and Beta believe that they can effectively represent their respective clients. (D) No, because Alpha and Beta sometimes share fees.

C Beta may not take the case even if Alpha and Beta believe that they can effectively represent their respective clients, and even if all parties give informed consent, confirmed in writing. The key issue is whether Alpha and Beta are considered a "firm" for purposes of the imputed disqualification rule with respect to conflicts of interest. Because lawyers in a firm are usually treated as a single unit for conflict of interest purposes, different lawyers in the same firm must not represent opposing parties in a civil case. [ABA Model Rule 1.10(a)] Relevant factors in determining whether lawyers who share office space are deemed a firm include whether they: hold themselves out to the public as a single unit, frequently consult and assist each other, refer cases to each other, work jointly on cases, and have access to each other's files. All of these factors are present in this case. Thus, Alpha and Beta are deemed a firm, and Beta is disqualified from accepting the employment because Alpha's disqualification is imputed to her.

An attorney is a voting member of the legislation committee of a consumer-based law reform group that drafts and advocates the passage of proposed statutes on food safety. The law reform group is currently debating a draft statute that sets quality and safety standards for growth hormones administered to chickens, turkeys, and other poultry. The attorney is also engaged in the private practice of patent law. She regularly represents a biotechnology firm. Using the techniques of genetic engineering, the biotechnology firm invents, develops, and sells a variety of patented growth hormones. The attorney herself has obtained patents on some of these hormones for the biotechnology firm. If enacted into law, the law reform group's proposed statute on poultry hormones could materially increase the biotechnology firm's hormone sales because it is the only firm whose hormones would meet the statute's quality and safety requirements. Would it be proper for the attorney, as a member of the law reform group's legislation committee, to participate in the debate on, and to cast her vote on, the proposed statute? (A) No, because the statute could materially benefit the biotechnology firm. (B) No, because the attorney may not serve as a member of the law reform group while representing the biotechnology firm. (C) Yes, provided that she informs the legislation committee that she represents an unnamed client whose interests could be materially benefited by the statute. (D) Yes, provided that she informs the legislation committee that she represents the biotechnology firm, whose interests could be materially benefited by the statute.

C It would be proper for the attorney to participate in the debate and cast her vote on the proposed legislation, provided that she informs the committee that she represents a client whose interests could be materially benefited by the statute. A lawyer may participate in a law reform activity that will affect the interests of the lawyer's client. [ABA Model Rule 6.4] When a lawyer knows that a client will be materially benefited by the activity, the lawyer must disclose that fact, but she need not name the client. (A) is wrong because a lawyer is not prohibited from engaging in a law reform activity that might benefit her client. (B) is wrong because a lawyer is not prohibited from participating in a law reform activity, unless the participation would create an impermissible conflict of interest. [ABA Model Rule 1.7(a)] That is not the case here. A client who hires a lawyer does not thereby purchase the right to control the lawyer's views and activities in all contexts. [See ABA Model Rule 1.2(b)] The attorney may even advocate new legislation that she thinks is sound that would harm the biotechnology firm's sales. [See ABA Model Rule 6.4] (D) is wrong because the attorney need not disclose the name of her client; simply disclosing the fact of representation will inform the legislation committee of her possible bias.

A lawyer is the head of the in-house law department of a children's clothing company, which has its principal place of business within the state. Under state law, it is a felony to manufacture or sell children's sleepwear that is not fire retardant. The president of the company informed the lawyer in confidence that the company is stuck with a whole warehouse full of children's pajama fabric that does not meet the state's fire standards, and that to avoid financial disaster, the company will use the fabric to make children's sleepwear and take its chances on legal liability. The lawyer was unable to convince the president to change his mind; she then raised the issue with the company's board of directors, which ratified the president's decision. Will the lawyer be subject to discipline if she resigns as house counsel and reports the matter to the appropriate state law enforcement authorities? (A) Yes, because the lawyer is required to preserve the corporation's confidential information even after she resigns. (B) Yes, because there is no adequate reason for permissive withdrawal on these facts. (C) No, because the lawyer is entitled to reveal this type of confidential information. (D) No, because the lawyer's duty to preserve confidential information ceases with her resignation as house counsel.

C The lawyer will not be subject to discipline for reporting the matter to the appropriate authorities because she is entitled to reveal this type of confidential information. The lawyer may resign her in-house counsel position because the company's board insists on following a course of action that is both repugnant and criminal. [ABA Model Rule 1.16(b)(2), (4)] The children's sleepwear is likely to cause substantial bodily harm or even death. Because an attorney is entitled to reveal confidential information to the extent she reasonably believes necessary to prevent reasonably certain death or substantial bodily harm, the lawyer may report this matter to the appropriate authorities. [ABA Model Rule 1.6(b)(1)] In addition, if the highest authority for an organization fails to take appropriate action regarding a violation of law, then a lawyer for the organization may report the relevant information to an appropriate person outside of the organization, if the lawyer reasonably believes that reporting is necessary to prevent substantial injury to the organization. This is true even if the information would otherwise be protected by the duty of confidentiality (which is not the case here because the sale of the fabric is likely to cause substantial bodily harm). [ABA Model Rule 1.13(c)] Here, the sale of the fabric is a violation of law, and selling the fabric could result in substantial injury to the organization if the fabric catches fire. (A) is wrong because the lawyer is entitled to reveal confidential information to prevent reasonably certain death or substantial bodily harm or to prevent substantial injury to the organization. (B) is wrong because the repugnance and criminality of the proposed conduct are both sufficient grounds for permissive withdrawal. [ABA Model Rule 1.16(2), (4)] (D) is wrong because the duty of confidentiality continues even after the termination of the attorney-client relationship. [See comment 18 to ABA Model Rule 1.6]

A bank operates a professional referral hotline for its depositors. Any bank depositor who needs to find a physician, lawyer, accountant, dentist, or the like can telephone the hotline and obtain a free referral from lists of professionals compiled by the bank. The lists are limited to professionals who maintain an average balance of at least $10,000 in an account at the bank, but the professional does not pay a fee to the bank for receiving a particular referral. A lawyer keeps $10,000 on deposit with the bank for the express purpose of being included on its lawyer referral list. Is this arrangement proper? (A) Yes, because the bank is functioning in the role of a lawyer referral service. (B) Yes, because neither the bank's depositors nor the professionals pay a fee for referrals. (C) No, because the lawyer is required to keep $10,000 on deposit to be included on the list. (D) No, because this arrangement constitutes an association with a nonlawyer for the practice of law

C The arrangement is not proper because the lawyer is required to keep $10,000 on deposit to be included on the list. A lawyer may not give "anything of value" to a person for recommending the lawyer's services. [ABA Model Rule 7.2(b)] The bank benefits in many ways by increasing the amount of its deposits; for example, its deposits determine how much it can lend to borrowers. Thus, obtaining deposits from lawyers is of value to the bank, and that is one reason it has devised the referral scheme. (A) is wrong because although a lawyer may pay the usual charges of a not-for-profit or qualified lawyer referral service [ABA Model Rule 7.2(b)], banks operate for profit, and there is no indication that the bank has been approved by the appropriate regulatory authority as a qualified lawyer referral service. (B) is wrong because the lawyer is giving something of value for the referrals, as explained above, even though there is no fee for individual referrals. (D) is wrong because this arrangement does not constitute an improper partnership or association with a nonlawyer for the purpose of practicing law. A lawyer's professional association with a nonlawyer is improper if the nonlawyer: (i) owns an interest in the practice; (ii) is an officer or director of a business involving law practice; or (iii) has the right to control the lawyer's professional judgment. [ABA Model Rule 5.4(d)] None of these is the case here; the bank is acting solely as a referral agent, and has nothing to do with the operation of the lawyer's practice.

An attorney worked at the United States Department of Labor and was responsible for compiling certain corporate safety records into an annual report containing the accident statistics. The report is used internally and in discussions with companies, but it is not distributed to the general public. However, a person may obtain a copy of the report by filing a formal request under the Freedom of Information Act. During the last three years, Company A has had more accidents than any of the other reporting companies. Six months ago, the attorney left the Labor Department and took a job with a private law firm. Recently, a person came to the attorney seeking representation in a suit against Company A for injuries he sustained while working at Company A's factory. The attorney agreed to represent the client. Is the attorney subject to discipline? (A) Yes, because he obtained relevant information about Company A while working as a government attorney. (B) Yes, because the attorney did not obtain the consent of the Department of Labor. (C) No, because the information is available by formal request under the Freedom of Information Act. (D) No, if the attorney does not use the information obtained while employed as a government attorney to the material disadvantage of Company A.

C The attorney is not subject to discipline for taking the case because the relevant information he obtained while working as a government attorney is not confidential. The general rule is that a government lawyer who receives confidential government information about a person must not later represent a private client whose interests are adverse to that person, if the information could be used to the material disadvantage of that person. [ABA Model Rule 1.11(c)] The rule covers only "confidential" information, which means information that the government is prohibited from revealing or has a privilege not to reveal, and which is not otherwise available to the public. Here, because the information is available under the Freedom of Information Act, it is not confidential. In fact, any attorney representing the client could obtain the information; thus, the attorney is free to use it.

An attorney is a partner in a four-partner law firm. A client entrusted $40,000 to the attorney, instructing the attorney to hold it in safekeeping for a few days and then to use it as the down payment on a piece of lakefront property. The attorney promptly deposited the money in his law firm's law office account, a special bank account that the firm uses to pay the office rent, pay staff salaries, advance litigation expenses on behalf of clients, and the like. A few days later, when it was time to make the down payment, the attorney discovered that one of his law partners had made two large withdrawals from the law office account, reducing the account balance far below the $40,000 needed for the client's down payment. The client's attorney was unable to come up with other money to make up the difference, and the client thus lost the chance to buy the lakefront property. Is the attorney subject to civil liability to the client for mishandling the money? (A) No, because it was the attorney's law partner, not the attorney, who made the two large withdrawals that made it impossible to come up with the down payment. (B) No, because the legal ethics rule governing safekeeping of clients' funds is for professional disciplinary purposes only; it is not intended as a standard for civil liability purposes. (C) Yes, because the attorney mishandled the money and is therefore civilly liable to the client for breach of his fiduciary duty. (D) Yes, because an attorney is strictly liable to a client for harm incident to the disappearance of money that the client has entrusted to the attorney.

C The attorney is subject to civil liability for mishandling the client's money because he breached his fiduciary duty to the client. When the attorney received the $40,000 from the client, he should have deposited it in a client trust account, not in the account that the law firm uses for office expenses. The attorney's failure to put the money in the correct account was a breach of fiduciary duty that can result in both professional discipline under ABA Model Rule 1.15(a) and in civil liability under the principles expressed in the Restatement.

An attorney and a licensed real estate developer, a nonlawyer, created a partnership to serve people who want to invest in commercial real estate. The real estate developer finds promising commercial real estate projects, brings together groups of investors, and works with local planning authorities to gain approval for the projects. The attorney drafts the legal documents for the projects, assists the investors with the legal technicalities, advises the investors on their tax liabilities, and does whatever legal work the investors need in connection with management and operation of the projects. The attorney and the real estate developer charge the investors a single fee for their work, and they divide the partnership profits 50%-50%. Is the attorney subject to discipline? (A) No, provided the investors give informed consent to the potential conflicts of interest, and such consent is confirmed in writing. (B) No, because the real estate developer does only development work, and the attorney does only legal work. (C) Yes, because the attorney and the real estate developer are partners in the business. (D) Yes, because she is aiding the real estate developer in the unauthorized practice of law.

C The attorney is subject to discipline because she and the real estate developer are partners in the business described in the question. A lawyer is prohibited from entering into a partnership with a nonlawyer if any of the partnership activities constitutes the practice of law. [ABA Model Rule 5.4(b)] The rationale and social policy behind this Rule have been sharply questioned, but the ABA has not abandoned its traditional distrust of partnerships with nonlawyers. [See Hazard & Hodes, §45.7] (A) is wrong because although there are potential conflicts here in that the attorney appears to work partly for the investors and partly for the developer and herself in putting the real estate projects together, informed consent, confirmed in writing, by the investors will solve these conflict issues. In any event, the attorney is still subject to discipline for entering into the partnership with a nonlawyer. (B) is wrong because the division of responsibility does not solve the partnership with a nonlawyer problem. (D) is wrong because the developer is not engaging in activities that could be construed as practicing law. All of the legal work (i.e., work calling for the professional judgment of a lawyer) is done by the attorney.

An attorney and her client endured a stormy attorney-client relationship until the attorney finally withdrew due to the client's repeated refusals to pay the attorney's fee bills. At the end of the relationship, the client owed the attorney more than $10,000. The client said he would not pay because the attorney's legal services were "defective." In a final effort to avoid having to sue the client for the unpaid fees, the attorney proposed a settlement agreement to the client. Under the proposed agreement, the attorney would accept $4,000 as full payment, reserving the right to sue the client for the other $6,000 if the client filed a State Bar disciplinary complaint against the attorney or filed a legal malpractice action against the attorney. The client signed the settlement agreement without consulting outside counsel, and the attorney did not suggest that he should consult outside counsel before signing it. Is the attorney subject to discipline for entering into the settlement agreement with the client? (A) No, because the attorney brought about an amicable settlement of the fee dispute with the client. (B) No, because there was a good faith dispute between the attorney and the client about the quality of the attorney's services and the amount of fees due. (C) Yes, because the attorney did not advise the client to seek outside counsel before entering into the settlement agreement. (D) Yes, because the attorney compromised a potential malpractice claim by the contract with her client.

C The attorney is subject to discipline because she did not advise the client to seek outside counsel. A lawyer may not settle a legal malpractice claim or potential claim with an unrepresented client or former client without first advising that person in writing to seek outside legal advice about the settlement and giving the person a reasonable chance to obtain such advice. [ABA Model Rule 1.8(h)(2)] Although the client has apparently not made a formal claim of malpractice here, he has asserted that the attorney's services were "defective," and that is regarded as sufficient to bring Rule 1.8(h)(2) into play. The settlement agreement in this question also gives the client an incentive not to report a lawyer's misconduct to the bar. Some state bars have found similar settlement agreements improper because they frustrated the bar's efforts at self-regulation and could be prejudicial to the administration of justice. [See Arizona State Bar Op. 91-23 (1991)] (A) is wrong because although lawyers are urged to settle fee disputes amicably [comment 9 to ABA Model Rule 1.5], this particular settlement agreement involves both a fee dispute and a malpractice claim. (B) is wrong because the dispute about the quality of the attorney's services is what causes the problem here, as explained above. (D) is wrong because it is too broad. A lawyer may settle a malpractice claim if the claimant is independently represented or if the lawyer advises the claimant in writing that he should seek independent legal advice before entering into the settlement. [ABA Model Rule 1.8(h)(2)]

An attorney was appointed by the court to defend a client at his criminal trial for second degree murder. The attorney started interviewing potential witnesses. When she interviewed the client's landlord, the landlord said that on the night of the murder, the client came home very late and was wearing a shirt covered with blood. The landlord died before trial without speaking to state authorities. Which of the following best states what the attorney should do with respect to the information she has learned from the landlord? (A) The attorney should voluntarily reveal the information to the prosecutor prior to trial because the death of the landlord has made it impossible for the prosecutor to obtain the information in any other way. (B) The attorney should urge the client to allow her to reveal the information to the prosecutor, and if the client refuses, the attorney should withdraw. (C) The attorney should keep the information in confidence unless the client authorizes her to reveal it, even though the death of the landlord has made it impossible for the prosecutor to obtain the information in any way other than from the attorney. (D) The attorney should use her own best judgment about how to treat the information; it is neither privileged nor confidential because it was not given to her by her client or by an agent of her client.

C The attorney should keep the information in confidence unless the client authorizes her to reveal it, even though the death of the landlord has made it impossible for the prosecutor to obtain the information other than from the attorney. The attorney obtained this information from the landlord in the course of representing her client; therefore, it is subject to the attorney's duty of confidentiality. Absent the consent of the client, an attorney must not reveal any information relating to the representation of the client. [ABA Model Rule 1.6]

For many years, a tax attorney has handled all of the tax work for his client, a sculptor. One evening, the sculptor invited the attorney to his studio to discuss some tax returns that had to be filed the next day. In the studio, the attorney saw a small sculpture that would be perfect for his office. At the close of their tax discussion, the attorney offered to buy the sculpture for $10,000, its approximate fair market value. The sculptor told the attorney that it was not for sale. In due course, the attorney sent the sculptor a bill for a $750 fee for the tax work. A few days later, the small sculpture was delivered to the attorney's office with a note from the sculptor, saying that he hoped the sculpture would satisfy the recent bill, and he wanted the attorney to have the sculpture as a token of his gratitude for the excellent tax advice. Would the attorney be subject to discipline for accepting the small sculpture from the sculptor? (A) Yes, because the gift is of significant monetary value. (B) Yes, because the value of the sculpture is far out of proportion to the $750 worth of work the attorney did for the sculptor. (C) No, because the attorney did not solicit the gift. (D) No, because the $10,000 is only an approximation of market value.

C The attorney would not be subject to discipline for accepting the sculpture because he did not solicit the gift. Although ABA Model Rule 1.8(c) prohibits a lawyer from soliciting a substantial gift from a client when the lawyer is not related to the client, it does not prohibit a lawyer from accepting an unsolicited gift from a client, even if the gift is substantial (although the gift may be voidable for undue influence). Moreover, comment 6 to ABA Model Rule 1.8 states that a lawyer may accept a gift from a client if the transaction meets general standards of fairness. Here, the attorney did not solicit the gift, and there are no facts to suggest undue influence or unfairness. Thus, the gift is proper.

An attorney is defending her client in a civil fraud case in which it is relevant to know what advice the client received in confidence from an independent certified public accountant. The jurisdiction has no evidentiary privilege for confidential communications between accountants and their clients. The accountant telephoned the attorney and asked how he should respond to the plaintiff's lawyer's request to speak with him privately about the case. Reasonably believing that the accountant would not be harmed by refusing to talk informally with the plaintiff's lawyer, the attorney responded that if the plaintiff's lawyer subpoenaed him to testify, then he must do so, but encouraged him not to talk to the plaintiff's lawyer about the case unless under subpoena. Was the attorney's advice to the accountant proper? (A) No, because the advice the accountant gave the client was not protected by an evidentiary privilege. (B) No, because the attorney interfered with the plaintiff's access to evidence. (C) Yes, because the accountant acted as the client's agent in rendering accounting advice to the client. (D) Yes, because it was improper for the plaintiff's lawyer to seek a private discussion with the accountant about the case.

C The attorney's advice to the accountant was proper because the accountant acted as the client's agent in rendering accounting advice. An attorney may request that someone other than a client refrain from voluntarily giving relevant information to another party if the person is a relative or agent of the client and the attorney reasonably believes that the person's interests will not be adversely affected by refraining from giving the information. [ABA Model Rule 3.4(f)] Here, the attorney reasonably believed that refusing to talk informally with the plaintiff's counsel would not harm the accountant, and the accountant was the client's agent in rendering the accounting advice. (A) is wrong because it is the accountant's agency, not privilege, that affects the propriety of the attorney's advice. The lack of an evidentiary privilege does not give the plaintiff's counsel a right to talk to the accountant informally if the accountant chooses not to. (B) is wrong because the attorney did not interfere with the plaintiff's access to evidence; if the plaintiff wants to know what the accountant has to say, he can simply take the accountant's deposition. (D) is wrong because it was not improper for the plaintiff's counsel to attempt to talk informally with a third-party witness such as the accountant.

Continuously since 1910, the law firm of Alpha & Beta has practiced under that name. The founders of the firm are long dead. No partner named Beta now practices with the firm. Two partners named Alpha were practicing with the firm, but one recently left because she was appointed to the state supreme court. May the firm continue to use the name Alpha & Beta? (A) No, because no partner named Beta now practices with the firm. (B) No, because one partner named Alpha left the firm to enter public service. (C) Yes, unless the firm name would be misleading. (D) Yes, even if the firm name will mislead some prospective clients.

C The firm may continue to use the name Alpha & Beta if it is not misleading. ABA Model Rule 7.5(a) permits a firm to practice under a trade name, provided that the trade name is not misleading in violation of ABA Model Rule 7.1. (A) is wrong because a firm may continue using the name of a deceased partner. [Comment 1 to ABA Model Rule 7.5] (B) is wrong because the person who left was not a name partner. Generally, when a name partner enters public service and is not in private practice for a substantial period, the firm must cease using that person's name. [ABA Model Rule 7.5(c)] Here, however, the Alpha surname refers to the deceased founder. If, however, the use of the Alpha name in the firm name would mislead potential clients (e.g., by making them think they could gain an advantage in the state supreme court by hiring that firm), then continued use of the name would violate the ethics rules. [See ABA Model Rule 7.1] (D) is wrong because the rules on firm names are subject to the more general provisions on misleading communications.

A law professor was selected as the neutral arbitrator of a boundary line dispute between an elderly couple and the couple's next-door neighbors. The law professor decided the matter in favor of the elderly couple. Shortly thereafter, the law professor quit his teaching position and entered private law practice. The elderly couple's next-door neighbors brought suit to have the arbitration award set aside. The elderly couple asked the law professor to represent them in the suit. If the law professor takes the case, will he be subject to discipline? (A) No, because serving as the elderly couple's lawyer is consistent with his decision as arbitrator in their favor. (B) No, because by seeking to hire the law professor, the elderly couple is deemed to have consented to the conflict of interest. (C) Yes, because his earlier service as neutral arbitrator creates a conflict of interest. (D) Yes, because there is reasonable ground to doubt his impartiality in the case.

C The law professor would be subject to discipline for representing the elderly couple because his earlier service as neutral arbitrator creates a conflict of interest. A lawyer must not represent a private client in a matter in which the lawyer has earlier participated personally and substantially while serving as an arbitrator. [ABA Model Rule 1.12(a)] (A) is wrong because the consistency of his position does not solve the conflict of interest. (B) is wrong because it does not go far enough. Informed consent, confirmed in writing, by both landowners would solve the conflict, but consent by the elderly couple alone will not suffice (and their consent was not confirmed in writing). (D) is wrong because it invokes a nonexistent rule; unlike judges, lawyers are not expected to be impartial. Here, the law professor would be acting as an advocate, not as a judge or arbitrator; thus, he should be partial to his client. The issue here is not partiality but conflict of interest.

A lawyer was assigned by the court to defend an indigent teacher at her murder trial. The jury convicted the teacher, and she was sentenced to 40 years in prison. The lawyer's court appointment expired at the end of the trial, but he promised the teacher that he would represent her without cost in taking an appeal from her conviction. The lawyer advanced $350 on the teacher's behalf to cover the expenses of the appeal, knowing that the teacher would probably not be able to pay him back. While the appeal was pending, the teacher wrote the manuscript for a book about life in a women's prison. She hired the lawyer to negotiate a contract with a publisher to have the book published, and in return for the contract work, she promised to pay the lawyer 30% of the royalties from her book. Is the lawyer subject to discipline? (A) Yes, because he entered into a literary rights contract with his client while her appeal was still pending. (B) Yes, because he advanced appeal expenses for his client, knowing that she probably could not pay him back. (C) No, unless 30% of the book royalties is unreasonably high for the contract negotiation work. (D) Yes, because he agreed to represent his client pursuant to a court appointment and therefore is not entitled to book royalties.

C The lawyer is not subject to discipline unless 30% of the book royalties is unreasonably high for the contract negotiation work. Here, the lawyer has, in essence, agreed to negotiate the publication contract in return for a contingent fee. Like all other fees, a contingent fee is subject to the general requirement of reasonableness. [ABA Model Rule 1.5] (A) is wrong because this is not the kind of literary rights contract that is prohibited. A lawyer must not acquire literary or media rights to a story concerning the lawyer's representation of a client until after the legal matter is entirely concluded. [ABA Model Rule 1.8(d)] Here, the client is the author, and the book is about her life in prison, not about her case or her lawyer's representation of her. A lawyer may represent a client in a transaction concerning literary property in which the lawyer's fee consists of a share of the ownership of the property, provided that the arrangement complies with the general rules about attorneys' fees and does not give the lawyer a proprietary interest in the subject of litigation. [Comment 9 to ABA Model Rule 1.8] (B) is wrong because a lawyer may advance litigation expenses for a client, even though he is aware that she probably cannot pay him back. ABA Model Rule 1.8(e)(2) permits the lawyer simply to pay the litigation expenses for an indigent client, even without the pretense of calling it an advance. (D) is wrong because the literary rights contract was a separate representation from, and did not concern the story of, the murder trial.

Two sisters are partners in a bakery. Their partnership agreement says that they will share the work and the profits equally. They are very close, but they constantly bicker—each claims that the other is taking an unfair share of the profits and shirking on the work. Six months ago, they hired a lawyer to act as a third-party neutral, to help them resolve their differences once and for all. At the outset, the lawyer explained that he would be strictly neutral between them; he would not be representing either one, and neither of them would be entitled to the protections afforded by an attorney-client relationship. After a long series of meetings with them (sometimes separately, sometimes jointly), the lawyer proposed a solution. The sisters liked his solution, reduced it to writing, and signed it, vowing to end their bickering forever. Six months later, the feud erupted again, worse than ever. One of the sisters asked the lawyer's law firm to represent her in a lawsuit against her partner-sister, seeking to declare the partnership at an end and to bar her partner-sister from entering the bakery premises. Which of the following is correct? (A) The lawyer is subject to discipline for his failed effort to serve both sisters when their interests were patently in conflict. (B) It would be proper for the lawyer to represent the sister in the lawsuit as she requested, even without the informed consent of her partner-sister. (C) The lawyer's law firm partner may represent the sister in the lawsuit as she requested, but only if her partner-sister is notified in writing, and only if the lawyer is timely screened and does not share in the fee earned in the lawsuit. (D) The lawyer's law firm partner would be subject to discipline for representing the sister in the lawsuit as she requested, even if the lawyer is timely screened and does not share in the fee earned in the lawsuit.

C The lawyer was not representing either sister; rather, he was acting as a third-party neutral to help them resolve their differences. ABA Model Rule 2.4 permits a lawyer to serve in that role. ABA Model Rule 1.12(c) permits screening to avoid a conflict in this situation. Therefore, (A) and (D) are incorrect. (B) is incorrect because, when a lawyer has served as a third-party neutral between two conflicted parties, he cannot later represent one of the parties in that matter, unless both parties give informed consent, confirmed in writing. [ABA Model Rule 1.12(a)]

The state bar association has established a peer counseling program whereby lawyers who are addicted to alcohol or other drugs can receive confidential counseling from other lawyers. The bar association's ethics rule on confidential information provides that communications between the counselor lawyer and the counseled lawyer are to be treated just like confidential communications between an attorney and client. A lawyer is addicted to alcohol and is receiving peer counseling under the program from another lawyer. The lawyer is a large, strong man, and his addiction has made him subject to periodic fits of physical violence. This afternoon, during their peer counseling session, the lawyer told his peer counselor that his client had refused to pay the fees he owes, and that he intended to punch out the client the next time he got roaring drunk. From working with the lawyer over an extended period, the peer counselor believes that he may really do it. May the peer counselor disclose the lawyer's statement to the client and the police? (A) No, unless the lawyer consents. (B) No, unless the peer counselor is certain that the lawyer will carry out his threat. (C) Yes, even if the lawyer objects. (D) Yes, because he is serving as a peer counselor, not a lawyer.

C The peer counselor may disclose the statement even if the lawyer objects. The state ethics rule on confidentiality treats communications between a lawyer and his peer counselor just like communications between an attorney and a client. If the counselor had heard one of her clients make this threat, she could have warned the police and the intended victim. An attorney may reveal confidential information to the extent she reasonably believes necessary to prevent reasonably certain death or substantial bodily harm. [ABA Model Rule 1.6(b)(1)] Thus, the peer counselor may warn the client and the police. (A) is wrong because the peer counselor may act to prevent the lawyer from causing substantial bodily harm. The lawyer's consent is not necessary. [ABA Model Rule 1.6(b)(1)] (B) is wrong because if an attorney reasonably believes that her client (or anyone else) is about to inflict substantial bodily harm on someone, she may take steps to prevent it, even if she is not certain that the client (or other person) will do it. (D) is wrong because the state ethics rule on confidentiality equates the peer counselor relationship with the relationship between an attorney and client; thus, the ability to disclose is the same.

A family-owned chemical company receives information that some of its sales associates may have entered into price fixing agreements in violation of federal antitrust laws. The shares of the chemical company are not publicly traded and the company is not subject to the jurisdiction of the Securities and Exchange Commission. The general counsel of the corporation hires an outside antitrust lawyer to investigate the matter. The antitrust lawyer and her associate investigated the matter and discovered that some of the sales associates had indeed entered into agreements that could make the corporation civilly and criminally liable under the antitrust laws. The antitrust lawyer reported these findings and her antitrust advice in a confidential letter addressed jointly to the company's chief executive officer and the general counsel. The general counsel wrote back, asking the antitrust lawyer to stand by to defend the company if needed. Months went by, and the antitrust lawyer heard nothing more. Her associate grew restless, and without telling the antitrust lawyer, he told a friend in the Justice Department what the company's salespeople had done. The Justice Department began a price fixing investigation of the company and its competitors. Which of the following is correct? (A) The antitrust lawyer is subject to discipline for failing to report the company's situation to the antitrust enforcement authorities in the Justice Department, but her associate's conduct was proper. (B) The conduct of both the antitrust lawyer and the associate was proper, and neither of them will be subject to civil liability if the company sues them for legal malpractice. (C) The associate is subject to discipline for tipping off the Justice Department, but the antitrust lawyer's conduct was proper. (D) Neither the antitrust lawyer nor the associate is subject to discipline, but both of them may be subject to civil liability if the company sues them for legal malpractice.

C This question is governed by ABA Model Rule 1.13, not by the SEC's regulations under the Sarbanes-Oxley Act, because the company is not publicly owned and is not subject to the jurisdiction of the SEC. The associate is subject to discipline for tipping off the Justice Department because he violated the duty of confidentiality imposed by ABA Model Rule 1.6(a). When the antitrust lawyer and the associate investigated the price fixing rumor at the request of the company's general counsel, they were operating under ABA Model Rule 1.13(d), which applies to lawyers who are hired "to investigate an alleged violation of law" or to "defend an organization" or its people against a claim arising out of an alleged violation of law. That means that ABA Model Rule 1.13(c) does not apply, and they must not report to outsiders about what they find

A lawyer was a widely admired, highly compensated trial attorney in solo practice. He represented clients in all types of civil and criminal litigation, mostly in high-profile cases that drew a lot of media attention. The governor of the state where the lawyer practiced had been harshly criticized for appointing appellate judges who lacked significant experience as trial counsel. Hoping to silence his critics, the governor appointed the lawyer to serve out the remaining seven years of a recently deceased supreme court justice's 12-year term. After the seven years, the lawyer can run for election to a new 12-year term. Before taking the oath as judge, the lawyer sold his entire law practice—books, client files, office lease, furniture, and goodwill—to another attorney. The lawyer gave appropriate advance notice to the clients, and the purchasing attorney covenanted that he would not raise their legal fees. A few years later, one of the cases that the lawyer transferred to the purchasing attorney came before the state supreme court on appeal. Which of the following propositions is false? (A) The lawyer's sale of his law practice was proper. (B) The purchaser's covenant not to increase the fees paid by the lawyer's clients was proper. (C) The lawyer must disqualify himself from the case involving his former client. (D) The lawyer may participate in the decision of the case involving his former client, provided that all of the other supreme court justices give their informed consent.

D (D) is false. CJC Rule 2.11(C) explains remittal of a judge's disqualification, and remittal requires the agreement of all of the parties and their lawyers, not the other justices. (A) is true. ABA Model Rule 1.17 permits the sale of an entire law practice, or an area of practice, subject to some conditions, all of which are met here. (B) is true. ABA Model Rule 1.17(d) provides that when a practice is sold, the fees charged to the clients cannot be increased by reason of the sale. (C) is true. CJC Rule 2.11(A)(6)(a) says that a judge must disqualify himself if he previously served as a lawyer in the matter.

A full-time trial judge, in addition to her judicial work, is the chief executive officer of a corporation that is closely held by the judge and her three brothers. The corporation owns and operates a nursing home. Because of strong anti-homosexual religious beliefs on the part of residents, the nursing home does not employ gays and lesbians. The judge's responsibilities for the corporation do not interfere with her judicial duties. Is it proper for the judge to continue as chief executive officer of the corporation? (A) Yes, because a judge is only prohibited from associating with an organization that practices invidious discrimination on the basis of race, sex, religion, or national origin. (B) Yes, because the management of the family-owned business does not take so much time that it interferes with the judge's judicial duties. (C) No, because a judge is not allowed to serve as an officer, director, manager, general partner, advisor, or employee of a business entity. (D) No, because the nursing home practices employment discrimination against gays and lesbians.

D A judge may not be affiliated with an organization that practices invidious discrimination on the basis of race, sex, gender, religion, national origin, ethnicity, or sexual orientation. [CJC Rule 3.6] The corporation excludes gays and lesbians who are otherwise qualified, making it improper for the judge to continue as chief executive officer.

A defendant asked a lawyer to defend him in a criminal case in which the defendant was charged with running a gambling operation. The defendant was known in the community as a wealthy person, but one who seldom kept his word and seldom paid his bills. The lawyer agreed to do the necessary work for a flat fee of $5,000, which was reasonable in light of the difficulty of the case and the number of hours required. However, the lawyer required that the defendant pay $3,000 in advance. When the defendant protested that he did not have that amount available in ready cash, the lawyer accepted the defendant's full-length fur coat in lieu of the cash advance. The fair market value of the coat is $3,000, and the lawyer agreed to return it upon payment of his fee. Their agreement was reduced to writing and signed by both parties. After the lawyer had put in considerable time in preparing the case for trial, the defendant fired the lawyer for no good reason and refused to pay him anything for the work already done. Assuming the reasonable value of the lawyer's services to date is $4,000, which of the following statements is correct? (A) The lawyer is subject to discipline for demanding that the defendant pay $3,000 in advance, before any legal services had been rendered. (B) The lawyer is subject to discipline for taking the coat in lieu of cash as an advance on legal fees. (C) The lawyer is not subject to discipline because the client had a reputation for not paying his bills. (D) The lawyer is not subject to discipline because all of his described conduct was proper.

D A lawyer may require his fee to be paid in advance and may accept property in return for services, provided it does not involve a proprietary interest in the subject of litigation. [Comment 4 to ABA Model Rule 1.5] Thus, the lawyer here has done nothing wrong. (A) is incorrect because, as discussed above, a lawyer may take an advance. (B) is incorrect because the coat is not the subject of litigation; thus, accepting the coat as payment was proper. (C) states the correct conclusion but is based on a faulty rationale—the propriety of the lawyer's conduct does not depend on the client's reputation for paying bills.

A patent attorney focuses her practice on patents that involve genetically engineered medicines. Representatives of a bioengineering firm had a preliminary conversation with the attorney about representing the bioengineering firm in a patent infringement action against a pharmaceutical corporation. The attorney had never represented either company previously. The bioengineering firm's representatives talked to the attorney for more than an hour about the bioengineering firm's patent and about the pharmaceutical corporation's supposedly infringing product. This conversation covered only public information, nothing confidential. The bioengineering firm's representatives detected a distinct lack of enthusiasm from the attorney, and they ended the conversation cordially but without hiring her. In due course, the bioengineering firm hired a different patent attorney and sued the pharmaceutical corporation for patent infringement. The pharmaceutical corporation hired the attorney as defense counsel in the infringement case. The bioengineering firm's attorney promptly made a motion in the trial court to disqualify the attorney because of her earlier conversation with the bioengineering firm's representatives. Is the attorney subject to disqualification? (A) Yes, because the bioengineering firm had previously consulted the attorney on the same matter. (B) Yes, because the infringement suit is substantially related to the earlier conversation between the attorney and the bioengineering firm's representatives. (C) No, because the bioengineering firm was never the attorney's client. (D) No, because the prior conversation between the attorney and the bioengineering firm's representatives did not involve confidential information.

D ABA Model Rule 1.18 provides that a lawyer must not use or reveal confidential information of a prospective client. [See also ABA Model Rule 1.6] Here, the information communicated to the patent attorney was not confidential; thus, the patent attorney's undertaking representation of the pharmaceutical corporation did not create a concurrent conflict of interest—there is no significant risk that the representation of the pharmaceutical corporation would be materially limited by the patent attorney's responsibilities to the bioengineering firm. Consequently, the patent attorney may continue to represent the pharmaceutical corporation. (A) is wrong because it does not matter that the bioengineering firm previously consulted the patent attorney on the same matter if the patent attorney did not obtain any confidential information that would limit her representation of the pharmaceutical corporation, and she does not breach any duty owed to the bioengineering firm. (B) is wrong for the same reason as (A)—even if the infringement suit is substantially related to the patent attorney's conversation with the bioengineering firm's representatives, unless the patent attorney obtained confidential information from the bioengineering firm, she may represent the pharmaceutical corporation. (C) is not as good as (D) because (C) is general, while (D) is specifically on point. Here, the patent attorney did not get any material confidential information from the bioengineering firm's representatives during the preliminary conversation, but if she had, she would be subject to disqualification as defense counsel, even though the bioengineering firm never became the patent attorney's actual client.

A man walked into a lawyer's office and explained that he is a dealer of illegal drugs, that one of his sales associates has just been arrested, and that he would like the lawyer to defend the sales associate. The drug dealer offered to leave a $10,000 retainer comprised of a shoebox of $100 bills. The lawyer was uncertain about getting involved with the drug dealer and especially about accepting a large sum in cash from the dealer. The lawyer told the dealer that he would think about his request and would let him know later that day. When the drug dealer left the office, the lawyer telephoned his friend and mentor, a retired judge. The lawyer told the judge the entire story, including the dealer's name, and asked the judge whether it would be ethical to defend the sales associate and accept the dealer's cash. Is the lawyer subject to discipline for telling the judge the whole story? (A) Yes, because the attorney-client privilege forbids the lawyer from revealing what the drug dealer told him in confidence. (B) Yes, because the solo practitioner revealed the drug dealer's name. (C) No, because the lawyer's prospective client was the accused sales associate, not the drug dealer. (D) No, because the ethical duty of confidentiality has an exception that allows a lawyer to reveal confidential information to obtain legal ethics advice.

D ABA Model Rule 1.6(b)(4) and comment 9 explain that a lawyer may reveal information that would otherwise be confidential if the lawyer's purpose is to obtain legal advice about complying with the legal ethics rules.

A private adoption agency handles over 65% of all private adoptions in the state in which the agency is located. The agency provides each set of prospective adoptive parents with a list of lawyers whom the agency recommends and tells them that it is in their best interest to obtain a lawyer experienced in adoptions. The agency has investigated each lawyer on its list to ensure the lawyer's experience and reputation for honesty and ethical behavior. To further protect prospective adoptive parents, the agency also requires each lawyer on the list to agree not to withdraw from any case where he has been retained by parents adopting through the agency. A lawyer who has handled many private adoptions and is highly regarded as an honest and competent lawyer would like to be placed on the agency's list. Would it be proper for the lawyer to sign the agreement and have himself included in the list? (A) Yes, because the list is a valuable service, and people often do not know where to find a lawyer qualified to handle a matter such as a private adoption. (B) Yes, because the lawyer does not give anything of value to the agency in exchange for being placed on the list. (C) No, because the agency is acting as the lawyer's agent, and a lawyer may not use an agent to contact persons in a manner that would be unethical solicitation if done by the lawyer. (D) No, because the agreement allows a third party to exercise influence over the lawyer-client relationship.

D It would not be proper for the lawyer to have his name included on the agency's list because the agreement allows a third party to exercise influence over the lawyer-client relationship. A lawyer must not allow a person who recommends, employs, or pays her for serving a client to direct or regulate the lawyer's professional judgment. [ABA Model Rule 5.4(c)] The agency is, in effect, recommending the lawyer, and the restriction on withdrawal in the form agreement clearly interferes with the lawyer's professional judgment. (A) is wrong because the good intentions behind the list do not remove the interference with the lawyer's professional judgment. Likewise, (B) is wrong because, even if the lawyer does not give the agency anything of value, the arrangement is still improper because of the restriction on withdrawal. (C) is wrong because the list does not amount to improper solicitation.

A solo practitioner is one of only three lawyers in a small town. The solo practitioner is presently defending a client in a criminal action for assault and battery. This morning one of the solo practitioner's regular clients, a gas and grocery store, asked the solo practitioner to sue the same client to recover a past due amount on a gasoline and grocery charge account. Would it be proper for the solo practitioner to represent the gas and grocery store in the charge account case? (A) No, because it is presumed that a lawyer obtains confidential information in the course of representing a client. (B) No, unless the other two lawyers in town are disqualified from representing the gas and grocery store. (C) Yes, because there is no substantial relationship between the charge account case and the assault and battery case. (D) Yes, if both the client and the gas and grocery store consent after full disclosure of the conflict, and such consent is confirmed in writing.

D It would not be proper for the solo practitioner to represent the grocery store unless both the client and the grocery store give informed consent, confirmed in writing, to the representation. A lawyer who is presently representing a client in one litigation matter should not simultaneously oppose that client in a different litigation matter, even if the two matters are unrelated. [ABA Model Rule 1.7(a) and comment 6] The purpose of the rule is to avoid putting the client into the difficult position of treating the lawyer simultaneously as friend and foe. The conflict can be solved only by informed consent, confirmed in writing, from both the client and the grocery store. (A) is wrong because the rule prohibiting the representation applies regardless of whether the solo practitioner has obtained relevant confidential information. (B) is wrong because the conflict is not obviated by the shortage of legal talent in the small town. (C) is wrong because the Rule applies even if the two cases are unrelated.

A young associate was assisting a senior partner in writing the reply brief in an appeal for one of the partner's clients. In doing the legal research, the associate discovered a recent case from the controlling jurisdiction that had not been cited in the adversary's brief. In the associate's opinion, the case was directly opposed to the position of the partner's client. The associate asked the partner about citing it in the reply brief, but the partner explained that, in his view, the case was not directly on point and did not have to be cited. The associate and the partner argued back and forth at some length and finally decided to submit the question to one of the other senior partners in the firm for a fresh view. That partner sided with the other partner, and the reply brief was filed without mentioning the case. May the associate write a short letter to the appellate court and the adversary lawyer, explaining his position and enclosing a copy of the case? (A) Yes, because the associate had a duty to call the case to the court's attention. (B) Yes, because the associate must not allow another person to interfere with his professional judgment. (C) No, because the associate must not communicate with a court ex parte about the merits of a pending case. (D) No, because the associate should abide by the partner's resolution of the matter.

D The associate should abide by the partner's resolution of the matter. A subordinate lawyer does not violate the Rules of Professional Conduct by acting in accordance with a supervisor's reasonable resolution of an arguable question of professional duty. [ABA Model Rule 5.2(b)] Here, it seems clear that the question was arguable because the third attorney called in to determine the relevance of the case also felt it was not on point. (A) is wrong because the associate only has a duty to call the case to the court's attention if the case is directly on point. That is a debatable question, and the associate's supervisors have determined the case is not directly on point. Thus, the associate need not reveal the case. (B) is wrong because this is not the situation intended to be addressed by the rule against allowing a third party to influence the lawyer's judgment, which usually arises when a third party pays the lawyer's fees to represent another. Of course, a subordinate lawyer should be influenced by his supervisor. That is not an excuse for clearly unethical conduct, but on a debatable issue, such as the one presented here, the subordinate lawyer is free to defer to the supervisor's judgment. (C) is wrong because in most jurisdictions a lawyer may communicate in writing with the court about the merits of a pending case if he sends a copy to opposing counsel. This communication is not considered ex parte. [See Restatement §113, comment c]

Two years ago, an attorney represented his client when he sold his property. Unbeknownst to the attorney, the client made some fraudulent statements to the buyer about the value of some mineral deposits on the property. The buyer recently discovered the fraud and is now in the attorney's office threatening to immediately file a civil fraud suit against both the client and the attorney. The buyer accuses the attorney of engineering the fraud and helping his client carry it out. The only way that the attorney can convince the buyer that he had no part in the fraud is to tell the buyer a fact that the client disclosed to him in the deepest confidence when he was working on the property transaction. May the attorney disclose the fact without the consent of the client? (A) No, if doing so will harm the client. (B) No, because doing so would breach his duty of confidentiality to the client. (C) Yes, but only after the buyer files the civil fraud suit against him. (D) Yes, even if doing so will subject his client to civil or criminal liability.

D The attorney may reveal the confidence even if doing so will subject his client to civil or criminal liability. A lawyer may disclose a client's confidence "to establish a defense to a criminal charge or civil claim against the lawyer based upon conduct in which the client was involved . . . ." [ABA Model Rule 1.6(b)(5)] Although the lawyer must wait until the assertion of misconduct arises, he need not await the filing of a formal charge or complaint. The lawyer may defend himself by responding directly to a third party who has made such an assertion. [See Restatement §64, comment c] (A) is wrong because the lawyer may disclose the fact even if doing so harms the client. (C) is wrong because the lawyer need not wait for the complaint to be filed, as explained above. (B) is wrong because it ignores the self-protection exception to the general rule of confidentiality.

A worker sued his employer, claiming that he was permanently and totally disabled due to a back injury he suffered on the job. The employer's lawyer strongly suspected, but had no proof, that the worker continued his hobby of skydiving after the alleged back injury. In due course, the employer's lawyer met with the worker's lawyer for a settlement discussion. The employer's lawyer told the worker's lawyer that they had movies of the worker jumping out of an airplane two weeks after his purported injury. The worker's lawyer excused herself to make a telephone call to the worker. When she asked the worker whether he had been skydiving after the accident, he admitted that he had. With the consent of their respective clients, the two lawyers then settled the case for $400. Is the employer's lawyer subject to discipline? (A) No, because bluffing is an accepted tactic in settlement negotiations between lawyers. (B) No, because the employer's lawyer's bluff successfully unmasked a fraudulent claim. (C) Yes, because it was improper to pay $400 to settle a fraudulent claim. (D) Yes, because the employer's lawyer lied about having movies.

D The employer's lawyer is subject to discipline because he lied about having movies. When dealing on behalf of a client with a third person, a lawyer must not knowingly make a false statement of law or material fact. [ABA Model Rule 4.1(a)] The employer's lawyer knew that he had no movies; his statement to the worker's lawyer was a bald lie. (A) is wrong because the employer's lawyer's statement was a knowing misrepresentation of material fact, not the kind of puffery that is tolerated in settlement negotiations. [See comment 2 to ABA Model Rule 4.1] (B) is wrong because the ends do not justify the means. (C) is wrong because the employer consented to the $400 settlement. Given the employer's risk of going to trial in a case where the plaintiff is claiming total and permanent disability due to a back injury, the small settlement was not unreasonable.

A lawyer in a partnership died, leaving his daughter, a doctor, as his sole heir. Under the partnership agreement, the firm plans to make the following payments to the deceased lawyer's daughter: $100,000, which represents the decedent's share of the firm's assets, as measured by his capital contribution; $45,000, which represents the decedent's share of fees that had been earned but not collected from clients at his death; and a $125,000 death benefit, representing a percentage of the decedent's earnings the year prior to his death, and payable in 12 monthly installments. Under the Model Rules, which of the following represents the most that the firm may properly pay to the decedent's daughter? (A) $100,000 for the decedent's share of the firm's assets. (B) $145,000 for the decedent's share of the firm's assets and his share of uncollected fees. (C) $170,000, which represents the death benefit and the decedent's share of uncollected fees. (D) $270,000, which includes the decedent's share of the firm's assets, his share of uncollected fees, and the death benefit.

D The firm may pay all of the money as planned. Even though the decedent's daughter is a nonlawyer, the firm may make certain kinds of payments to her from money originally earned as legal fees. [ABA Model Rule 5.4(a)(1)] The $100,000 is a proper payment because it reflects the decedent's share of the capital assets of the firm. The $45,000 is a proper payment because the fees it represents had been earned, albeit not collected, at the time the decedent died. The $125,000 is a proper payment because it is a reasonably computed death benefit payable over a reasonable period of time.

An elderly widower has one living child, a daughter. The widower's main asset is a 51% partnership interest in a wealthy real estate syndicate that owns and operates mobile home parks throughout the state. The daughter's husband is an attorney. One of the husband's regular clients asks the husband to represent him in negotiating the sale of 3,000 acres of roadside property to the real estate syndicate. The real estate syndicate is represented by its own lawyer in the matter. May the husband represent his regular client in a sale with the real estate syndicate? (A) No, even if the client gives informed consent, confirmed in writing. (B) No, because to do so would create an appearance of impropriety. (C) Yes, because the husband has no significant personal interest in the real estate syndicate. (D) Yes, but only if the client gives informed consent, confirmed in writing.

D The husband may represent his regular client if the client gives informed consent, confirmed in writing, to the representation. His wife is likely to inherit her father's interest in the real estate syndicate. That gives the husband a personal interest in the real estate syndicate, albeit an attenuated interest. If the husband is to represent his regular client in selling land to the real estate syndicate, he must first disclose his personal interest to the client. If the client gives informed consent, confirmed in writing, then the husband may represent the client. [ABA Model Rule 1.7(b)] (A) is wrong because informed, written consent will solve the potential conflict of interest. (B) is wrong because informed consent, confirmed in writing, will solve the conflict problem. Furthermore, the "appearance of impropriety" is not a basis for discipline or disqualification under the ABA Model Rules. This is an outdated concept from the old ABA Model Code. (C) is wrong because the husband's personal interest is significant, even though it is remote.

An attorney is admitted to practice only in State A, where he specializes in securities and real estate finance law. In that role, the attorney advised his client that the law of State B did not require the client to include information about certain mineral rights in a disclosure statement that the client had to file in State B in order to sell some real estate limited partnership interests to State B citizens. Acting on the attorney's advice, the client did not disclose the information and did sell partnership interests to State B citizens. Later, the attorney became a full-time trial court judge in State A. Later still, State B brought a criminal action against the client for failing to disclose the mineral rights information in his State B disclosure statement. One of the client's defenses is that he lacked the necessary criminal intent because he was acting in good faith based on the advice of his counsel, the attorney who is now a judge. The client needs the judge's testimony to prove that the judge did indeed advise him that he was not required to disclose the mineral rights information. The judge, in State A, is beyond the subpoena power of the State B court. May the judge voluntarily testify on behalf of the client? (A) No, because judges are disqualified from serving as witnesses in criminal cases. (B) No, because he is not admitted to practice in State B, and his testimony about State B law would be inadmissible. (C) Yes, because a judge may testify as a witness, except in his own court or one under its appellate jurisdiction. (D) Yes, because his testimony would concern the giving of the advice, not his client's character.

D The judge may testify because he is testifying to facts, not the defendant's character. CJC Rule 3.3 prohibits a judge from testifying voluntarily as a character witness, but it says nothing about serving as an ordinary fact witness. (A) is wrong because there is no such rule. Judges are not disqualified from testifying in criminal cases. (B) is wrong because an attorney need not be admitted to practice in State B in order to advise a client about State B law. Even if that were untrue, the judge's testimony would still be admissible as evidence of the client's lack of criminal intent. (C) is wrong because it states a nonexistent rule. While a judge is not competent to be a witness at a trial over which he himself is presiding [seeFed. R. Evid. 605], no rule forbids a judge from serving as an ordinary fact witness in a case that is pending before a different judge in his own court or a court that is under his court's appellate jurisdiction.

An inventor asked a patent lawyer to represent him in obtaining a patent on a new computer technique for predicting the growth patterns of tumors in the human body. The lawyer informed the inventor that he had never worked on that kind of patent application before, and that he would have to do extensive background research on the patentability of computer techniques. The lawyer will be able to use the knowledge that he gains through the research to serve other clients who wish to obtain patents for all manner of other computer techniques. The lawyer offered to do the work for the inventor for his standard hourly rate, but the inventor proposed instead to assign the lawyer a 10% interest in the patent, if and when it was issued. The lawyer agreed to do the work on that basis, and he and the inventor entered into an appropriate written fee agreement. The lawyer did the work; the patent was ultimately issued and proved so valuable that the lawyer was able to sell his 10% interest for $9.7 million (which was a reasonable fee for the work performed). Is the lawyer subject to discipline? (A) Yes, because he acquired a proprietary interest in the subject of the representation. (B) Yes, because it is unreasonable to charge one client for background research that will be used to earn fees from other clients. (C) No, because the inventor agreed to the fee arrangement. (D) No, because $9.7 million is not an unreasonably high fee for the work that the lawyer did.

D The lawyer is not subject to discipline because $9.7 million is within the bounds of reason as a fee for the work the lawyer did. [See ABA Model Rule 1.5(a)] Among the various factors that point to the reasonableness of the lawyer's fee are: the novelty and difficulty of the patentability issue, the fact that the inventor was the one who suggested the fee arrangement after having been offered a standard hourly fee, the value of the result that the lawyer obtained for the inventor, and the contingent nature of the arrangement, which imposed a high risk on the lawyer. (A) is wrong because a lawyer is prohibited from acquiring a proprietary interest in the subject of litigation he is conducting [ABA Model Rule 1.8(i)], and obtaining a patent is not litigation. Even if this were a litigation case, the contingent fee exception to the rule would apply. [ABA Model Rule 1.8(i)(2)] (B) is wrong because a fee that is otherwise reasonable does not become unreasonable simply because the lawyer can use the knowledge gained to earn fees from other clients. (C) is wrong because the mere fact that the client agreed to the fee arrangement does not by itself make the fee reasonable. Many factors, including the time, labor, and skill required to do the job, are considered in determining whether the fee is reasonable. [ABA Model Rule 1.5(a)]

A client hired a lawyer to do the legal work in connection with a complex public securities offering. The lawyer agreed to do the work for a set hourly fee. The lawyer did a great deal of legal research, prepared numerous memoranda of fact and law, and drafted most of the documents needed for the public offering. At that point, the client became angry with the lawyer for no apparent reason and fired him. The client paid the lawyer at the agreed rate for the work the lawyer had done and demanded that the lawyer turn over to him the papers that the lawyer had prepared, including the legal and fact memoranda and the document drafts. What papers must the lawyer turn over to the client? (A) Only the document drafts, but not the legal and fact memoranda. (B) Only the legal and fact memoranda, but not the document drafts. (C) None of the papers, because the client fired the lawyer. (D) All of the papers, even though the client fired the lawyer.

D The lawyer must turn over all of the papers to the client. When a lawyer is fired, he must return all "papers and property to which the client is entitled." [ABA Model Rule 1.16(d)] In this case, the client is entitled to all the papers the lawyer has prepared. Under the law of many states, an attorney can assert a lien on client papers in her possession to secure the payment of her fee, but here the client has paid the lawyer for all the work the lawyer did. (A) is wrong because the lawyer must turn over the memoranda as well as the other documents. (B) is wrong because the lawyer must give the client the document drafts as well as the memoranda. (C) is wrong because the fact that the lawyer was fired, even without cause, does not in any way change the lawyer's duty to give the client all of the papers.

A building contractor and his lawyer met with a landowner to negotiate a contract for construction of an office building on the landowner's property. The contractor, the lawyer, and the landowner were the only persons present at the meeting. Ultimately, the three of them worked out a written agreement, and the contractor commenced work. However, it soon became apparent that the building site required far more preparation work than the contractor had contemplated when he agreed to the contract price. The contractor and the landowner argued about who had to pay for the additional site preparation. One important issue is whether the landowner made certain oral representations to the contractor during the contract negotiating session that the lawyer attended. The contractor contends that the landowner did make the representations, while the landowner contends that he did not. The lawyer was present during the entire negotiating session, and she is virtually certain that the landowner did not make the representations. The contractor refused to proceed with construction until the landowner paid for the extra site preparation. The landowner then sued the contractor for specific performance of the construction contract. The contractor asked the lawyer to represent him as trial counsel. The lawyer should: (A) Agree to serve as trial counsel for the contractor because the contractor is entitled to the counsel of his choice. (B) Agree to serve as trial counsel for the contractor because she can refuse to testify if she is called as a witness by the landowner. (C) Decline to serve as trial counsel for the contractor because a lawyer is not allowed to testify in a manner that is prejudicial to her client. (D) Decline to serve as trial counsel for the contractor because she can foresee that she will be called as a witness.

D The lawyer should decline to serve as trial counsel because she can foresee that she will be called as a witness. A lawyer must not act as an advocate at a trial at which the lawyer is likely to be a necessary witness. [ABA Model Rule 3.7(a)] The lawyer was the only person other than the parties present at the negotiating session. The landowner will almost certainly call the lawyer as a witness. Because the lawyer can foresee at the outset that she will likely be called as a witness, she should decline to serve as trial lawyer for the contractor, even if the contractor is willing to consent to the conflict of interest. [ABA Model Rule 1.7(a); comment 6 to ABA Model Rule 3.7] (A) is wrong because a person's choice of counsel is limited by the restraints imposed by the ethics rules. The client may choose the lawyer, but the lawyer cannot ethically accept the employment. (B) is wrong because the lawyer cannot refuse to testify if the landowner calls her to the witness stand. (C) is wrong because if a lawyer is called to the witness stand and sworn to tell the truth, she must do so, even if it is prejudicial to her client.

A man alleges that a very wealthy actor punched him in the face. He contacted a lawyer about representing him in a civil action against the actor. After several lengthy discussions with the lawyer about the merits of the case, the man decided to employ another attorney instead. The actor was later charged with criminal assault in connection with this incident, and the trial was televised. As the lawyer was watching the trial, she was astonished when the man testified to facts that the lawyer knew from their previous discussions to be false. The lawyer sent a letter with a messenger over to the court to notify the court that the man had perjured himself. Were the lawyer's actions proper? (A) Yes, because her actions were necessary to prevent the man from perpetrating a fraud on the court. (B) Yes, because the man committed a criminal act by testifying falsely. (C) No, unless she sent copies of the letter to the prosecution and defense attorneys and they are given an opportunity to respond. (D) No, because the lawyer's information was gained during her discussions with the man.

D The lawyer's actions were not proper because her information was gained from a confidential lawyer-client communication. [See ABA Model Rule 1.6] It was both privileged and confidential, and could be disclosed only if one of the exceptions to the duty of confidentiality applies. None of the exceptions applies here. The fact that the man did not hire the lawyer to represent him does not affect her duty of confidentiality; he was seeking legal advice and representation when he spoke to her. [See ABA Model Rule 1.18(b)] (A) is wrong because there is no exception to the duty of confidentiality to prevent a fraud on the court when the lawyer is not appearing before the court. (B) is wrong because this too does not fit within any exception to the duty of nondisclosure. A lawyer may reveal confidential information to the extent she reasonably believes necessary to prevent, mitigate, or rectify substantial injury to the financial interests of another that is reasonably certain to result or has resulted from the client's commission of a crime or fraud, if the client has used the lawyer's services in furtherance of such crime or fraud. [ABA Model Rule 1.6(b)(3)] Here, the facts do not indicate that the man used the lawyer's services in furtherance of his perjurious testimony. Thus, although the perjury could result in substantial injury to the actor's financial interests, the lawyer is not permitted to reveal the confidential information. (C) is wrong because it states the rule for an ex parte communication to the judge by one of the parties. Here, the lawyer does not represent one of the parties. Furthermore, the information is confidential and cannot be disclosed to the disadvantage of the client regardless of whether the lawyer sends copies to all parties.

A public defender was assigned to represent a defendant at the defendant's preliminary hearing on a charge of kidnapping for ransom. Against the public defender's advice, the defendant testified on his own behalf at the preliminary hearing. The defendant was bound over for trial. At that point, the defendant's brother provided money to hire a private lawyer to represent the defendant, and the public defender was discharged. The defendant testified on his own behalf at the trial, and the matter concluded after the jury acquitted him. Later, in connection with his work on another matter, the public defender read the transcript of the defendant's trial. Based on information the public defender learned while representing the defendant, the public defender concluded that the defendant had committed perjury, both at the preliminary hearing and at the trial. However, the public defender does not believe that the defendant poses a danger to the community. May the public defender reveal the defendant's perjury? (A) Yes, the public defender may reveal the perjury committed at the defendant's preliminary hearing. (B) Yes, the public defender may reveal both instances of perjury (C) No, because the public defender does not believe that the defendant poses a danger to the community. (D) No, because disclosure would violate the public defender's duty of confidentiality.

D The public defender may not reveal the defendant's perjury because to do so would violate the public defender's duty of confidentiality. No exceptions to the confidentiality requirement apply to these facts. There is no indication that revealing the perjury is necessary to prevent reasonably certain death or substantial bodily harm. Also, there is no indication that the defendant's perjury is a crime that is reasonably certain to result in substantial injury to the financial interests of another, in furtherance of which the defendant has used the public defender's services. The obligation to reveal perjury under the Model Rules does not apply because that obligation ceases at the end of the proceedings, and both proceedings here have concluded.


संबंधित स्टडी सेट्स

CHAPTER 16 REGULATION OF FINANCIAL INSTITUTIONS

View Set

Honors World Geography Ch 9 Test 6

View Set

MSU anatomy chapter 26 pratice exam

View Set